Histology Krok 1

You might also like

Download as docx, pdf, or txt
Download as docx, pdf, or txt
You are on page 1of 103

Histology

Krok 1

Cytology 1

1. What is the limiting membrane of a cell?


A. *Plasmalemma
B. Glycocalyx
C. Protoplasm
D. Cristae
E. Ground substance

2. What is also called a low resistance junction?


A. Tight junction
B. *Gap junction
C. Junctional epithelium
D. Junctional complex
E. None of the above

3. What is the term for the general process that cells us to bring things into the cell?
A. *Endocytosis
B. Exocytosis
C. Pinocytosis
D. Phagocytosis
E. Active transport

4. Which one of the following transport processes require energy?


A. *Active transport
B. Passive transport
C. Facilitated diffusion
D. Simple diffusion
E. Filtration

5. Symport refers to the process of transporting


A. *two different molecules in the same direction
B. a molecule out of the cell
C. two different molecules in the opposite direction
D. a molecule into the cell
E. a molecule between the cytoplasm and the nucleus

6. Glycocalix of cell plasmolemma is damaged as a result of microbe toxins action. Which


membrane function of the cells is broken in the first place?
A. Trophic
B. Transporting
C. Respiratory
D. *Receptor
E. Protective

7. The photomicrography shows joining the epithelial cells with basal membrane. Name this type
of the epithelial attachment.
A. Simple junction
B. Nexus
C. Hemidesmosome
D. *Desmosome
E. Macula adherens
8. Golgi complex exports substances from a cell due to the fusion of the membrane saccule with
the cell membrane. The saccule contents flow out. What process is it?
A Exocytosis
B Endocytosis
C Active transport
D Facilitated diffusion
E All answers are false

9. As a result of activation of the ion channels of the external membrane, the rest potential of an
excitable cell has greatly increased. What channels were activated?
A Potassium channels
B Sodium channels
C Fast calcium channels
D Slow calcium channels
E Sodium and calcium channels

10. An electron micrograph shows a cell-to-cell adhesion consisting, in each cell, of an


attachment plaque. The intercellular space is filled with an electron-dense substance including
transmembrane fibrillar structures:
A Desmosome
B Synapse
C Tight junction
D Nexus
E Adherens junction

11. Which of the following best defines the term “exocytosis"?


a. The discharge of ions or small molecules from a cell by protein pumps in the cell membrane
b. The uptake of material at one domain of a cell’s surface and its discharge from the opposite
side of the cell
c. The process by which proteins move from one cytoplasmic compartment to another
d. *The discharge of proteins in cytoplasmic vesicles from a cell following fusion of the vesicles
with the plasmalemma
e. Diffusion of lipid-soluble molecules from a cell across the cell membrane

12. A 56-year-old man has been taking atorvastatin because of a poor lipid profile and a family
history of cardiovascular disease. The statin family of drugs enhances endocytosis of low-density
lipoprotein (LDL) from the blood. Endocytosis of LDL differs from phagocytosis of bacterial
cells in which of the following ways?
a. Use of membrane-enclosed vesicles in the uptake process
b. Coupling with the lysosomal system
c. Dependence on acidification
d. *Use of clathrin-coated pits
e. Use of hydrolases

13. A 42-year-old woman of Mediterranean descent presents with multiple oral blisters and a few
cutaneous blisters on her back and buttocks. The superficial bullae are fragile, some have
unroofed to form ulcerated lesions, and there is a positive Nikolsky sign. Blood tests reveal
antibodies to a subfamily of cadherins and immunohistochemical staining of a biopsy from the
oral mucosa shows distribution of the antigen throughout the epithelium. In what structures is the
defect that is causing this patient’s condition?
a. *Desmosomes
b. Tight junctions
c. Hemidesmosomes
d. Gap junctions
e. Reticular lamina
14. With a 5-year history of chronic respiratory infections, a 23-year old, non-smoking man is
referred to an otolaryngologist. A bronchial biopsy indicates altered structures in the epithelial
cells. Which of the following, if altered to reduce function, is most likely involved in this
patient’s condition?
a. Hemidesmosomes
b. *Cilia
c. Basolateral cell membrane folds
d. Microvilli
e. Tight junctions

15. Using immunohistochemistry, a population of cells is shown to be positive for the protein
connexin. From this we can infer that the cells are connected by what type of junction?
a. Tight (occluding) junctions
b. Zonula adherens
c. *Gap junctions
d. Hemidesmosomes
e. Desmosomes (macula adherens)

16. An individual genetically unable to synthesize normal occludin is likely to have epithelia
with defective regulation in which of the following?
a. *Material crossing the epithelium between the cells (paracellular movement)
b. Communication between cells
c. Attachment to the basement membrane
d. Strong attachment to neighboring cells
e. Movement of membrane proteins in the apical domains of cells

17. Cholesterol functions in the plasmalemma to


A. increase fluidity of the lipid bilayer.
B. * decrease fluidity of the lipid bilayer.
C. facilitate the diffusion of ions through the lipid bilayer.
D. assist in the transport of hormones across the lipid bilayer.
E. bind extracellular matrix molecules.

18. Which one of the following substances is unable to traverse the plasma membrane by simple
diffusion?
A. 02
B. N2
C. *Na+
D. Glycerol
E. C02

19. Symport refers to the process of transporting


A. a molecule into the cell.
B. a molecule out of the cell.
C. two different molecules in opposite directions.
D. *two different molecules in the same direction.
E. a molecule between the cytoplasm and the nucleus.

20. One of the ways that cells communicate with each other is by secretion of various molecules.
The secreted molecule is known as
A. a receptor molecule.
B. *a signaling molecule.
C. a spectrin tetramer.
D. an integrin.
E. an anticodon.
21. Examination of the blood smear of a young patient reveals misshapen red blood cells, and
the pathology report indicates hereditary spherocytosis. Defects in which one of the following
proteins cause this condition?
A Signaling molecules
B G proteins
C *Spectrin
D Hemoglobin
E Ankyrin

Cytology 2
1. Electron microscopic study of a cell revealed roundish bubbles confined by a membrane and
containing a lot of various hydrolytic enzymes. It is known that these organelles provide
intracellular digestion and protective functions. These elements are:
A *Lysosomes
B Centrosomes
C Endoplasmic reticulum
D Ribosomes
E Mitochondria

2. A 50-year-old woman had her tooth extracted. The tissue regenerated. Which of the following
organella are the most active during tissue regeneration?
A *Ribosomes
B Centrosomes
C Postlysosomes
D Agranular endoplasmic reticulum
E Lysosomes

3. Analysis of an electron diffraction pattern of a cell revealed mitochondrion destruction. This


might result in an abnormal course of the following cell process:
An *Oxidation of organic substances
B Nuclear division
C Crossingover
D Cleavage
E-

4. Formation of ribosome subunits in a cell was disturbed in course of an experiment (by means of
activated mutagenic factors). This will have an effect on the following metabolic process:
A *Protein biosynthesis
B Carbohydrate biosynthesis
C ATP synthesis
D Photosynthesis
E Biological oxidation

5. What are the folds on the inner mitochondrial membrane called?


A. Plasmalemma
B. Glycocalyx
C. Protoplasm
D. *Cristae
E. Ground substance

6. Which organelle is involved in lipid metabolism?


A. Rough endoplasmic reticulum
B. *Smooth endoplasmic reticulum
C. Lysosome
D. Golgi apparatus
E. Mitochondria

7. Which organelle contains detoxifying enzymes?


A. Ribosomes
B. *Peroxisome
C. Microfilaments
D. Centrioles
E. Nucleoli

8. Which of the following is NOT a membranous organelle?


A. *Microtubules
B. Lysosomes
C. Peroxisomes
D. Mitochondria
E. Endoplasmic reticulum

11. Which of the following is an organelle?


A. Pigment
B. Glycogen
C. Lipid
D. Secretory granules
E. *Mitochondria

13. Which organelle produces protein for export?


A. *Rough endoplasmic reticulum
B. Smooth endoplasmic reticulum
C. Lysosome
D. Golgi apparatus
E. Mitochondria

14. An animal had been intensively fed with carbohydrates. Histological examination of its liver
revealed a significant number of glycogen granules. Glycogen relates to the following group of
cell structures:
A *Trophic granules
B Secretory granules
C Excretory granules
D Pigment granules
E Special-purpose organelles

15. Low level of albumins and fibrinogen was detected in the patient's blood. Decreased activity
of what organelle of the liver hepatocytes can cause it?
A *Granular endoplasmatic net
B Agranular endoplasmatic net
C Mitochondrion
D Golgi complex
E Lysosomes

16. Ultramicroscopic examination of "dark" hepatocyte population in the cell cytoplasm detected
a developed granular endoplasmic reticulum. What function has this organelle in these cells?
A *Synthesis of blood plasma proteins
B Carbohydrate synthesis
C Deintoxicative function
D Bile production
E Calcium ion depositing
18. The prolonged influence of toxic substances on an organism resulted in considerable
reduction of the synthesis of proteins in hepatocytes. What organelle did suffer from the
intoxication more than others?
A *Granular endoplasmic reticulum
B Mitochondria
C Microtubules
D Lysosomes
E Golgi Complex

19. During histochemical research of hepatocyte in the cytoplasm of cell, it was found vesicles in
diameter of 0.05-1.5µm filled with enzymes of peroxide oxidation - catalase, peroxidase. What is
the name of organelle?
A *Peroxisome
B Lysosome
C Melanosome
D Liposome
E Phagosome

20. On the electronic photomicrograph of nervous cells of spinal ganglion, it was found
organelles, with cisterns, flattened in the central part and extended on the periphery. Also, small
vesicles were seen. How are these organelles called?
A *Golgi Complex
B Centriole
C Lysosomes
D Peroxisome
E Mitochondria

25. The electronic photomicrography shows the round structure by size 15-20 nm consisting of
big and small subunits. It is known that this organelle provides protein synthesis. Indicate it.
A. Microtubule.
B. Centrosome.
C. Peroxisome.
D. *Ribosome.
E. Mitochondrion.

26. The electronic photomicrography shows the round vesicles by size 0,2 - 0,5 mcm covered by
a membrane and filled by the matrix. The histochemical study of the matrix has found the
enzyme catalase. Name it.
A. Ribosomes.
B. Lysosomes.
C. *Peroxisomes.
D. Centrosomes.
E. Mitochondrion.

27. The system of the limited by biomembrane cavities was researched in cells which function is
defined by lipid and carbohydrate metabolism, detoxification and deposition of ion calcium.
Name this organelle.
A. Granular ER.
B. Golgi apparatus.
C. *Agranular ER.
D. Centrosome.
E. Mitochondria.
28. Ultramicroscopic study of the cell has shown well developed granular ER and Golgi
apparatus. Indicate specialization of this cell.
A. Phagocytosis.
B. The synthesis and secretion of lipids.
C. Absorption.
D. *The synthesis and secretion of proteins.
E. Accumulation and storage of the nutrients.

29. The Analysis ultrastructural constitution of the cells has shown it specialization to lipids and
cholesterol synthesis. The development which organelles most typifying for such cells?
A. Granular ER.
B. Golgi apparatus.
C. *Agranular ER.
D. Centrosome.
E. Mitochondria.

30. Which of the following has a rigid wall composed of 13 protofilament strands?
A. Microfilament.
B. Intermediate filament.
C. *Microtubule.
D. Myofilament.
E. Neurofilament.

Oval and round organelles with double wall are seen at the electron micrograph. The outer
membrane is smooth, the inner membrane folded into cristae contain enzyme ATP synthase.
These are:
A *Mitochondria
B Golgi complex
C Lysosomes
D Centrioles
E Ribosomes

A tissue sample of a benign tumor was studied under the electron microscope. A lot of small (15-
20 nm) spherical bodies, consisting of 2 unequal subunits were detected. These are:
A *Ribosomes
B Golgi complex
C Smooth endoplasmic reticulum
D Microtubules
E Mitochondria

Labeled amino acids alanine and tryptophan were injected to a mouse in order to study the
localization of protein synthesis in its cells. The labeled amino acids will be accumulated near
the following organelles:
A *Ribosomes
B Smooth endoplasmic reticulum
C Cell center
D Lysosomes
E Golgi apparatus

In course of practical training students studied a stained blood smear of a mouse with bacteria
phagocyted by leukocytes. What cell organelle completes digestion of these bacteria?
A *Lysosomes
B Mitochondrion
C Granular endoplasmic reticulum
D Golgi apparatus
E Ribosomes
An 18-year-old student presents with enlarged thyroid gland accompanied by accelerated
metabolism and increased heart rate. These signs can be observed during the hypersecretion of
thyroxin. What organelles of thyroid cells are primarily responsible for hormone production and
secretion?
A. *Golgi apparatus
B. Mitochondria
C. Ribosomes
D. Centrosomes
E. Lysosomes

Cells of healthy liver actively synthesize glycogen and proteins. What organelles are the most
developed in them?
A *Granular and agranular endoplasmic reticulum
B Cell center
C Lysosomes
D Mitochondria
E Peroxisomes

The cytochemical investigation revealed high content of hydrolytic enzymes in the cytoplasm.
This phenomenon indicates the activity of the following organelles:
A *Lysosomes
B Endoplasmic reticulum
C Mitochondria
D Polysomes
E Cell center

Cytology 3

17. The synthesis of histone proteins was artificially blocked in a cell. What structure of the cell
will be damaged?
A *Nuclear chromatin
B Nucleolus
C Golgi Complex
D Cellular envelope
E Nuclear envelope

Moving of the daughter chromatids to the poles of the cell is observed in the mitotically dividing
cell. On what stage of the mitotic cycle is this cell?
A *Anaphase
B Metaphase
C Telophase
D Prophase
E Interfase

RNA-polymerase B(II) is blocked due to amanitine poisoning (poison of death cup). It disturbs:
A *Synthesis of m-RNA
B Synthesis of t-RNA
C Reverse transcription
D Primers synthesis
E Maturation of m-RNA12

Examination of a patient revealed reduced contents of magnesium ions that are necessary for
attachment of ribosomes to the granular endoplasmic reticulum. It is known that it causes
disturbance of protein biosynthesis. What stage of protein biosynthesis will be disturbed?
A *Translation
B Transcription
C Replication
D Aminoacid activation
E Termination

While studying maximally spiralized chromosomes of human karyotype the process of cell
division was stopped in the following phase:
A *Metaphase
B Prophase
C Interphase
D Anaphase
E Telophase

Normal, actively dividing cells of human red bone marrow are analyzed. What number of cells’
chromosomes is typical for G1 period?
A *46
B 48
C 47
D 45
E 23

The life cycle of a cell includes the process of DNA autoreduplication. As a result of it
monochromatid chromosomes turn into bichromatid ones. What period of the cell cycle does this
phenomenon fall into?
A *S
B Go
C G1
D G2
E M1

It was proved that a molecule of immature mRNA (precursor mRNA) contained more triplets
than amino acids found in the synthesized protein. The reason for that is that translation is
normally preceded by:
A *Processing
B Initiation
C Reparation
D Mutation
E Replication

A specimen of an onion rootlet includes a cell in which the fully condensed chromosomes are
located in the equatorial plane making the monaster. What phase of the mitotic cycle is the cell
in?
A *Metaphase
B Early telophase
C Prophase
D Interphase
E Late telophase

In the life cycle of a cell during mitosis, a natural change in the amount of genetic material
occurs. The DNA doubles at the following stage:
An *Interphase
B Prophase
C Metaphase
D Anaphase
E Telophase
Students study the stages of gametogenesis. They analyze a cell with a haploid number of
chromosomes, where each chromosome consisting of two chromatids. The chromosomes are
located in the equatorial plane of the cell. Such a situation is typical of the following stage of
meiosis:
A *Metaphase of the second division
B Metaphase of the first division
C Anaphase of the first division
D Anaphase of the second division
E Prophase of the first division

At a certain stage of the cell cycle chromosomes reach cellular poles, undergo despiralization;
nuclear membranes are being formed around them; nucleolus is restored. What stage of mitosis
is it?
A *Telophase
B Prophase
C Prometaphase
D Metaphase
E Anaphase

Histological investigation of renal biopsy material taken from a patient with tuberculosis has
revealed chaotically located chromatin granules in the focus of caseous necrosis. These changes
are the result of:
A Karyorrhexis
B Karyolysis
C Karyopyknosis
D Mitotic activity of nuclei
E Apoptosis

Micromanipulator was used to extract one of the two centrioles of centrosome (cell center) from
hepatocyte (liver cell). What process will STOP OCCURRING in this cell?
A *Division
B Energy exchange
C Glycogen synthesis
D Protein biosynthesis
E Lipid synthesis

A patient has decreased concentration of magnesium ions that are required for ribosomes
connection to the granular endoplasmic reticulum. This condition is known to disturb the process
of protein biosynthesis. A disturbance occurs at the following stage:
A. *Translation
B. Transcription
C. Replication
D. Amino acids activation
E. Processing

492. During research on amniotic fluid, gotten by amniocentesis (puncture of the amniotic
cavity), it was discovered cells with sexual chromatin in their nuclei (Barr body). What can be
said about this?
A *Female sex of the fetus
B Male sex of the fetus
C Genetic violations in development of the fetus
D Trisomy
E Polyploidy

Binding of histone H1 proteins to importins is important for which of the following?


a. *Transport through the nuclear pores complexes
b. Properly directed vesicular transport through the Golgi apparatus
c. Transport from the granular part of the nucleolus
d. Further binding to the “linker DNA” and proper assembly of nucleosomes
e. Phosphorylation of cyclins

Which of the following is a region of chromatin that is well developed in large neurons active in
protein synthesis?
a. Heterochromatin
b.* The nucleolus
c. The Nissl substance (neuronal RER)
d. The Barr body
e. The nucleosome

5. Transitions in the cell cycle from one phase to the next are regulated by protein kinases whose
activity depends on what other proteins?
a. Tumor suppressors
b. *Cyclins
c. Actins
d. Lamins
e. Importins

6. Mitotic figures visible in a tissue section from the lining of the small intestine are most likely
to belong to which of the following categories?
a. Terminally differentiated cells
b. Partially differentiated cells
c. Blood cells
d. Stem cells
e. *Progenitor cells

7. Key differences between apoptotic and necrotic cell death include which of the following?
a. *Apoptotic cells do not release factors that induce inflammation.
b. Necrosis does not trigger inflammation.
c. Apoptosis does not utilize intracellular proteases.
d. Apoptosis usually follows lethal physical damage to a cell.
e. Necrosis is involved in the formation of some organs during embryonic development.

Embryology 1

471. Implantation process has two stages: adhesion and invasion. The morphological
manifestation of blastocyte adhesion is:
A *Attachment of blastocyte to the endometrium
B Destruction of endometrium epithelium
C Destruction of the connective tissue of the endometrium
D Destruction of endometrium vessels
E Formation of lacunes

472. For an unknown reason, the fertilization membrane of an embryo dissolved in the fallopian
tube in the first critical period. What complication of pregnancy is possible in this case?
A *Embryo implantation into the Fallopian tube
B Embryonic death
C Invagination of the blastocyst wall
D Return of blastocyst back to the ampullary portion of the tube
E Formation of two blastocysts

474. At a certain stage of development of a human embryo one can observe the formation of a
cavity in its structure, small light blastomeres on the periphery and large dark blastomeres at one
of the poles. The embryo at this stage of development is called:
A *Blastocyst
B Morula
C Zygote
D Gastrula
E Blastodisk

Microscopy of an oval cell, 150 microns in size, revealed the following: cytoplasm has yolk
inclusions but no centrioles. Name this cell:
A *Oocyte
B Leucocyte
C Myocyte
D Fibroblast
E Macrophage

491. It was found out that the human embryo is built up from two blastomeres. Name the place
of its localization under normal conditions of development?
A *Uterine tube
B Cavity of uterus
C Abdominal region
D Mucus envelope of uterus
E Ovary

493. On a scheme it is presented, the human embryo on one of the earliest stages of
development. What is it the stage?
A *Blastocyst
B Zigota
C Morula
D Gastrula
E Neurula

496. In the cavity of uterus it was found out that the human embryo is not attached to the
endometrium. To what stage of development does this embryo correspond?
A *Blastocyst
B Zigota
C Morula
D Gastrula
E Neurula

498. At the microscopic investigation of a female's internal reproductive organs which were
removed during operation, it was found an embryo which is built from two blastomeres. Name
the place of its localization under normal conditions of development?
A *Uterine tube, close to the ampulla
B Uterine tube, close to the uterine part
C Cavity of uterus
D Abdominal region
E Ovary

The process of splitting zygotes ends with the formation of blastules. What type of blastula is
characteristic for a human?
A *Blastocyst
B Celoblastula
C Discoblastula
D Amphiblastula
E Morula

Electron diffraction pattern of a spermatozoonclearlypresentsasheath- like structure localized in


the spermiocalyptrotheca and participating in dissolution of ovular membranes. Name
thisstructure:
A *Acrosome
B Centriole
C Microtubules
D Axoneme
E Segmented columns

A 17-year-old girl with a history of pelvic inflammatory disease presents at the emergency
department with severe pain in her lower right side that came on fairly quickly. Upon questioning
she replies that her last menstrual period was 6 weeks ago and that she has never missed a period
before. The doctor suspects she has an ectopic pregnancy and this is quickly confirmed by
ultrasound testing. The surgeon removes her right uterine tube which is inflamed, scarified, and
contains the implanted embryonic tissue in the region where fertilization normally occurs. Where
is this?
a. The uterine part of the oviduct
b. *The ampulla region with highly folded mucosa
c. The only oviduct region attached to the mesosalpinx
d. The infundibulum region with fimbriae
e. The isthmus region

A healthy woman, who had viral roseola during pregnancy, gave birth to a baby with a cleft lip
and cleft palate. The baby has a normal karyotype and genotype. This anomaly can be the result
of:
A. *Influence of teratogenic factor.
B. Gene mutation.
C. Chromosomal aberration.
D. Chromosomal mutation.
E. Combinative variability.

Two little bladders adjacent to each other (amniotic and yolk sac) can be seen in a 10-day
embryo specimen. What is the structure called that exists in the place of their contact?
A. Amniotic leg (crus).
B Bottom of amniotic sac.
C. Roof of yolk sac.
D. * Embryonic plate.
E. Extraembryonic mesoderm.

In the histological specimen is observed an extraembryonic organ that represents the bladder
linked with intestinal tube. Its wall is covered with epithelium internally and with embryonic
connective tissue from the outside. At the early stages of embryogenesis, it functions as
hematopoietic organ. The name of this organ is:
A. Amnion.
B. Allantois.
C. *Yolk sac.
D. Umbilical cord.
E. Uterine artery.

At the early stages of the human embryo development there springs up digital process from out
of the internal wall of primary gut rooting into the amniotic leg. What is the name of this
extraembryonic structure?
A. Yolk sac.
B. * Allantois.
C. Amnion.
D. Placenta.
E. Umbilical cord.

In the histological specimen of human fetus, a bladder linked with intestinal tube which is one of
the extraembryonic organs. Within its wall exist primary genital cells and primary erythrocytes
(megaloblasts). Define what this organ is.
A. *Yolk sac.
B. Allantois.
C. Placenta.
D. Umbilical cord.
E. Amnion.

In the course of experiment on an embryo of the frog, the external embryonic plate — ectoderm
was destructed. Which one of the following morphological structures won't be developed?
A. *Epidermis.
B. Somites.
C. Nephrotome.
D. Splanchnotome.
E. Myotome.

In a histological specimen is observed a hen embryo that is in mesoderm differentiation to


somites, segmental legs and splanchnotome. Which material mentioned below will the axial
skeleton is developed?
A. Myotome.
B. Dermatome.
C. Nephrotome.
D. Splanchnotome.
E. *Sclerotome.

The process cleavage of zygote comes to an end with formation of the blastula. What type of
blastula is specific for human?
A. Discoblastula.
B. Celloblastula.
C. * Blastocyst.
D. Amphiblastula.
E. Morula.

During the third week of embryogenesis, the central part of the epiblast (ectoderm) is sinked and
the neurolational process starts. In which direction will the remaining cells of ectoderm
differentiate?
A. Gut.
B. * Skin.
C. Somites.
D. Chord.
E. Yolk sac.

Embryology 2

469. A histological specimen of mandible of an embryo shows a tooth germ with the dental
papilla made up of small stellate basophilic cells. What tissue forms this part of the tooth germ?
A *Mesenchyme
B Epithelial
C Reticular
D Cartilaginous
E Osseous

473. By producing a number of hormones placenta plays a part of temporary endocrine gland.
What hormone may be detected in woman's blood on the third or the fourth day after begin of
implantation, that is used in medicine for early pregnancy detection?
A *Chorionic gonadotropin
B Somatostatin
C Progesterone
D Vasopressin
E Oxytocin
475. Microspecimen of a child's finger skin reveals subnormal development of epidermis. What
embryonic leaf was damaged in course of development?
A *Ectoderm
B Mesoderm
C Entoderm
D Mesenchyma
E Ectomesenchyma

476. During the embryogenesis of oral cavity the development of dental enamel was disturbed.
What source of dental development was damaged?
A *Epithelium
B Mesenchyma
C Mesoderma
D Dental saccule
E Dental papilla

477. During gastrulation the Hensen's node remained underdeveloped in the embryo. Which
axial organ will slow down its development?
A *Chord
B Neural crests
C Neural groove
D Neural tube
E Mantle layer of the neural tube

495. One of critical periods of human embryogenesis is implantation of embryo in the wall of
uterus during the 7th day. Which of the processes of gastrulation take place during this period?
A *Delamination
B Migration
C Epiboly
D Invagination
E Neurulation

497. At the ultrasound inspection of a pregnant woman, hydramnios (increase of amniotic fluid)
was diagnosed. The violation of the activity of which extraembryonic organ is connected with
this condition?
A *Amniotic envelope
B Chorion
C Placenta
D Vitelline sack
E Allantois

499. Study of the biopsy material of an embryo revealed a zone of developmental abnormality in
a somite. The zone was located close to the endoderm and the notochord. What formations may
have abnormal development in case of pregnancy continuation?
A *Skeletal tissues
B Genito-urinary system
C Skeletal striated muscle tissue
D Cardiac striated muscle tissue
E Fibrous connective tissue of skin

500. A histological specimen shows a transverse section of an organ, whose basis is formed of
mucous connective tissue, two arteries, and one vein. What organ is it?
A. Yolk sac.
B. Allantois.
C. *Umbilical cord.
D. Amnion.
E. Placenta.

A woman who was infected with toxoplasmosis during the pregnancy has born a child with
multiple congenital defects. This is a result of:
A *Teratogenesis
B Cancerogenesis
C Biological mutogenesis
D Chemical mutogenesis
E Recombination

Continuous taking of some drugs foregoing the pregnancy increase the risk of giving birth to a
child with genetic defects. What is this effect called?
A * Mutagenic effect
B Embryotoxic effect
C Teratogenic effect
D Fetotoxical effect
E Blastomogenic effect

Histological specimen of a 10-day human embryo represents 2 contacting sacs (amniotic and
yolk sacs). Specify the structure that separates the amniotic cavity from the yolk sac:
A *Embryonic shield
B Amniotic stalk
C Floor of the amniotic sac
D Roof of the yolk sac
E Extraembryonic mesoderm

A histological specimen shows significant amount of mucous connective tissue (Wharton’s


jelly), vessels, as well as residual yolk and allantois. Name this organ:
A *Umbilical cord
B Esophagus
C Ureter
D Urethra
E Vermiform appendix

An infant has been diagnosed with microcephaly. Doctors suspect that this brain disorder
developed due to the fact that the mother had been taking actinomycin D during her pregnancy.
What germinal layers have been affected by this teratogen?
A *Ectoderm
B Entoderm
C Mesoderm
D Entoderm and mesoderm
E All germinal layers

A newborn boy has been diagnosed with hydrocephalus. Doctors consider it to be caused by
teratogenic factors. What germ layers are affected by the teratogen?
A *Ectoderm
B All embryo germ layers
C Endoderm and mesoderm
D Endoderm
E Mesoderm

An 8-week-pregnant woman with acute respiratory disease and temperature rise up to 39, 0oC
has called in a doctor. The doctor insisted on her avoiding taking paracetamol, because in this
period of pregnancy there is a risk of its:
A *Teratogenicity
B Embryotoxicity
C Fetotoxicity
D Hepatotoxicity
E Allergenicity

At a certain stage of human ontogenesis, physiological bond occurs between circulatory systems
of the mother and the fetus. This function is being carried out by the following provisory organ:
A *Placenta
B Yolk sac
C Amnion
D Serous tunic
E Allantois

During gastrulation an embryo proceeds from histiotrophic to hematotrophic nutrition. What


provisory organ makes it possible at first?
A *Chorion
B Trophoblast
C Yolk sac
D Amnion
E Allantois

A microslide of the skin sample taken from the finger of a child shows that epidermis is
insufficiently developed. What germ layer was damaged in the process of embryo development?
A. *Ectoderm
B. Mesoderm
C. Endoderm
D. Mesenchyme
E. Ectomesenchyme

Neural crest cells give rise to which of the following?


A Dorsal horns of the spinal cord
B Adrenal cortex
C *Sympathetic ganglia
D Preganglionic autonomic nerves
E Somatic motor neurons

A 33-year-old woman with an average menstrual cycle of 28 days comes in for a routine Pap
smear. It has been 35 days since the start of her last menstrual period, and a vaginal smear
reveals clumps of basophilic cells. As her physician you suspect which of the following?
a. She will begin menstruating in a few days.
b. She will ovulate within a few days.
c. Her serum progesterone levels will be found to be very low.
d. *There will be detectable levels of human choriogonadotrophin (hCG) in her serum and urine.
e. She is undergoing early menopause.

143. During experiment a myotome of rabbit embryo was destroyed. Violation of development
of what structure will be caused by this manipulation?
A *Skeletal muscular tissue
B Axial skeleton
C Connective tissue of skin
D Smooth muscular tissue
E Serosa
Epithelial tissues and glands

31. Histological examination of a tissue sample revealed that the tissue had no blood vessels, and
the cells were packed tightly together making layers. Specify this tissue:
A *Epithelial
B Cartilaginous
C Osseous
D Nervous
E Muscular

32. There is a specimen of soft palate where both oral and nasal surfaces can be seen. It was
revealed that oral cavity had damaged epithelium. What epithelium is damaged?
A *Multistratal squamous nonkeratinizing
B Multistratal cubical nonkeratinizing
C Multistratal columnar nonkeratinizing
D Multistratal squamous keratinizing
E Multirowed ciliated epithelium

33. What type of epithelium forms the epidermis?


A. Simple squamous epithelium
B. Simple cuboidal epithelium
C. Simple columnar epithelium
D. *Stratified squamous epithelium
E. Pseudostratified epithelium

34. What type of tissue lines most of the gastrointestinal tract?


A. Simple squamous epithelium
B. Simple cuboidal epithelium
C. *Simple columnar epithelium
D. Stratified squamous epithelium
E. Transitional epithelium

35. What type of tissue forms the alveoli in the lung?


A. *Simple squamous epithelium
B. Simple cuboidal epithelium
C. Simple columnar epithelium
D. Stratified squamous epithelium
E. Pseudostratified epithelium

36. What type of epithelium is composed of flat cells?


A. Simple
B. Stratified
C. *Squamous
D. Cuboidal
E. Columnar

37. What do you call the simple squamous epithelium that lines the abdominal cavity?
A. Epithelioid tissue
B. *Mesothelium
C. Endothelium
D. Transitional
E. Pseudostratified

38. What type of epithelium is composed of cells which all touch the basement membrane
and is only one cell layer thick?
A. Stratified squamous epithelium
B. Transitional epithelium
C. Stratified cuboidal epithelium
D. *Pseudostratified epithelium
E. None of the above

39. Which of the following is NOT lined by a mucosa?


A. Genitourinary tract
B. *Pericardial cavity
C. Respiratory tract
D. Alimentary canal
E. All of the above are lined by a mucosa

40. What is a gland called if it has a branched duct?


A. Simple gland
B. *Compound gland
C. Tubular
D. Alveolar
E. Tubuloalveolar

41. What are finger like projections on the surface of some cells called?
A. *Microvilli
B. Stereocilia
C. Cilia
D. Keratinization
E. Both A and B

42. What cell surface modification is made of microtubules?


A. Microvilli
B. Stereocilia
C. *Cilia
D. Keratinization
E. Both A and B

43. A histological specimen of a mandibular gland shows an excretory duct. Mucous membrane
of the duct is lined with cubic epithelium whose cells have weakly developed organelles. What
excretory duct is it?
A *Intercalated
B Striated
C Interlobular
D Common excretory
E -

44. Epithelial cells are connected with each other by various contacts. What type of intercellular
contacts provides transport of ions and low-molecular substances from a cell in a cell?
A. Zonula occludens
B.Desmosome
C. Hemidesmosome
D. *Nexus
Е. All set forth above

45. In polar-differentiated columnar epithelium, which develops from intermediate mesoderm,


the part of cells in apical department carries structures containing axonema. Specify epithelium.
A. Intestine with a border
B. Glandular of stomach
C. *Ciliated of trachea
D. Ciliated of uterine tube
Е. Bordered of kidney tubules

47. On histological specimen of a wall of small intestine columnar epithelial cells with microvilli
on apical surface are visible. Epithelium carries out absorptive function. Specify epithelium.
А. Simple cuboidal
B. *Simple columnar with border
C. Mesothelium
D. Endothelium
Е. Simple columnar glandular

48. On histological specimen of trachea epithelium is seen, the most part of which cells has the
columnar form with cilia in apical of a part. Between them goblet cells, basal and intercalated are
situated. The nuclei of the listed cells are arranged in few rows. Specify epithelium.
A. Simple cuboidal
В. *Pseudostratified columnar ciliated
C. Mesothelium
D. Endothelium
Е. Simple columnar glandular

49. Epithelial tissue borders with connective tissue. What structure is located between them?
A. Аmorphous substance
B. Collagen fibers
C. Elastic membrane
D. Plasma membrane
Е. *Basement membrane

50. Epithelial tissues the various functions carry out. What from the listed functions is not
characteristic for them?
A. Secretory
B. Protective
C. *Trophic
D. Covering
Е. Contractile

51. The mucous layer of a stomach protects its wall from rough influence of food clods and
digesting action of gastric juice. To what morphological type of epithelial tissue does epithelium
of a stomach mucosa concern?
A. Simple squamous
B. Simple cuboidal
C. Simple columnar bordered
D. Pseudostratified ciliated
Е. *Simple columnar glandular

52. There is an ontophylogenetic classification of epithelial tissue created by N.G.Chlopin, in


which basis of the epitheliums origin is necessary. To what type epithelium, according to this
classification, does stratified squamous keratinized concern?
А. *Epidermal
B. Endodermal
C. Coelonephrodermal
D. Ependimoglyal
Е. Angiodermal

53. There is an ontophylogenetic classification of epithelial tissue created by N.G.Chlopin, in


which basis of the epitheliums origin is necessary. To what type epithelium, according to this
classification, does stratified transitional concern?
A. Epidermal
B. Endodermal
С. *Coelonephrodermal
D. Ependimoglyal
Е. Angiodermal

54. Easy trauma of a skin (scratch) at the child has disappeared in 10 days. Cambial elements of
which epidermis layer provided it reparative regeneration?
А. *Basal layer
B Covering layer
С. Granular layer
D. Keratinous layer
Е. Lucidum layer

55. Endocrine glands produce highly active substance - hormones. What from the listed
attributes are characteristic for endocrine glands?
А. *Secretion in blood
B. Secretion on a surface of a body
C. Secretion on a surface of organ
D. Secretion in a cavity of a body
Е. All is incorrect

56. Because of place of secretion glands are divided on endocrine and exocrine. Some
morphological features distinguish them also. What from the listed attributes is characteristic for
endocrine glands?
A. Presence of ducts
В. *Absence of ducts
C. Lobular structure
D. Trabecular structure
Е. All is incorrect

57. Because of place of secretion glands are divided on endocrine and exocrine. Some
morphological features distinguish them also. What from the listed attributes is characteristic for
exocrine glands?
A. *Presence of ducts
В. Absence of ducts
C. Lobular structure
D. Trabecular structure
Е. All is incorrect

58. The chemical composition of products secreting by exocrine glands can be various. How
differ exocrine glands depending on character of a secretion?
A. Serous
B. Mucous
C. Mixed
D. Sebaceous
Е. *All listed

59. Secretion is the process of formation and deliverance of the substances, synthesized by a cell.
It consists of several stages. What stages does secretory cycle consist of?
A. Absorption of initial substances
B. Synthesis and accumulation of a secret
C. Secretion
D. Restoration
Е. *All listed

60. The process of epithelium keratinization constitutes conversion of its alive epithelial cells to
cornified flakes - mechanically strong and chemically steady postcellular structures. Name the
basic processes occurring in cells during keratinization:
А. Change cell shape
B. Keratin formation
C. Organelles and nucleus destruction
D. Cytoplasm dehydration
E. *All listed

61. The patient has hyperkeratosis - condition of a skin, at which the process of cornified flakes
of epidermis removal is broken. Name the physiological mechanism of cells of a superficial layer
of stratified epithelium removal:
А. *Desquamation
B. Maceration
C. Migration
D. Autolysis
E. Apoptosis

62. On a scheme there is an exocrine gland which has unbranched excretory duct in which one
terminal portion is opened as one sac. What is the name of such gland according to
morphological classification of exocrine glands?
A * The simple alveolar unbranched
B The compound branched alveolar
C The simple branched tubular
D The compound unbranched alveolar
E The compound unbranched alveolar-tubular

63. In a patient with dry pleuritis during auscultation it was notices the noise of pleural friction
rub. What epithelium is damaged?
A * Simple squamous epithelium
B Simple squamous epithelium
C Simple cylindrical epithelium
D Transitional epithelium
E Stratified epithelium

64. After the prolonged inflammation of mucous membrane of nasal cavity in a patient changes
of epithelium had taken place. What epithelium did suffer?
A * Simple pseudostratified
B Simple squamous epithelium
C Stratified squamous epithelium
D Stratified cuboidal epithelium
E Stratified cylindrical epithelium

65. The internal envelope of vessels (intima) is lined by epithelium. Name it.
A *Endothelium
B Mesothelium
C Epidermis
D Transitional epithelium
E Pseudostratified epithelium

66. At the mechanical trauma of scrotum of a patient it was observed damage of epithelial lining
of testis. What epithelium was damaged?
A * Simple cuboidal
B Ciliated
C Simple columnar
D Two-layered
E Transitional

67. Patient addressed to the ophthalmologist with complaints about griping in eyes, which
happened after prolonged staying of patient in the field during a dustborne storm. A doctor
revealed superficial damages of external epithelium of cornea. What cells will provide
regeneration of the damaged epithelium?
A *Basal cells
B Cells of horny layer
C Cells of granular layer
D Cells of transparent layer
E Cells of superficial layer

A microslide contains the specimen of a gland composed of several secretory saccule-shaped


parts that open in the common excretory duct. What gland is it?
A * Simple branched alveolar gland
B Compound branched alveolar gland
C Simple unbranched alveolar gland
D Compound unbranched alveolar gland
E Simple branched tubular gland

Blood

68. In course of an operation on account of a granuloma in the area of the right upper incisor a
patient began to bleed. The hemorrhage was stopped just only 3 hours later. The patient's
anamnesis contains information about chronic lymphatic leukemia. What is the most probable
cause of hemorrhage?
A *Thrombocytopenia
B Thrombocytopathia
C Lymphocytosis
D Leukopenia
E Eosinophilia

71. Blood analysis of a 16-year-old girl suffering from the autoimmune inflammation of thyroid
gland revealed multiple plasmatic cells. Such increase in plasmocyte number is caused by
proliferation and differentiation of the following blood cells:
A *B-lymphocytes
B T-helpers
C Tissue basophils
D T-killers
E T-suppressors

73. An electronic microphotograph shows a macrophagic cell with erythrocytes at different


stages of differentiation located along its processes. This is the cell of the following organ:
A *Red bone marrow
B Thymus
C Spleen
D Tonsil
E Lymph node

74. Which leukocyte is the second most abundant in a peripheral smear of blood?
A. *Lymphocytes
B. Basophils
C. Neutrophil
D. Monocytes
E. Eosinophils

75. Which of the following is a granulocyte?


A. Lymphocyte
B. *Neutrophil
C. Monocyte
D. Erythrocyte
E. Thrombocyte

76. Which leukocyte has orange-pink granules?


A. Neutrophil
B. Lymphocytes
C. Monocytes
D. *Eosinophil
E. Basophils

77. What is another term for a red blood cell?


A. Thrombocyte
B. Monocyte
C. Lymphocyte
D. Basophil
E. *Erythrocyte

78. Which of the following is described as a "biconcave disc"?


A. Platelets
B. *Erythrocytes
C. Leukocytes
D. Monocytes
E. Eosinophils

79. Which is the largest leukocyte?


A. Neutrophil
B. Lymphocytes
C. *Monocytes
D. Eosinophil
E. Basophils

80. As a result of a road accident a 26-year-old man is in the torpid phase of shock. Blood count:
leukocytes - 3,210 9/l. What is the leading mechanism of leukopenia development?
A *Leukocyte redistribution in the bloodstream
B Leukopoiesis inhibition
C Faulty release of mature leukocytes from the bone marrow into the blood
D Leukocyte destruction in the hematopoietic organs
E Increased excretion of the leukocytes from the organism

81. Blood includes formed elements and plasma - liquid intercellular substance. Name parameter
estimating ratio of formed elements volume to volume of blood:
А. Hemogram
B. Leukogram
С. Myelogram
D. *Hematocrit
Е. ESR

82. Blood plasma contains important for vital functions components. In particular, in its
composition more 200 kinds of proteins are present. Which of listed proteins is lacking in
plasma?
А. Albumin
B. Globulin
С. Fibrinogen
D. *Hemoglobin
Е. Components of complement
83. The analysis of blood is widely used in practical medicine. Name the results of the analysis
indicating the contents of separate formed elements.
А. *Hemogram
B. Leukogram
С. Myelogram
D. Hematocrit
Е. ESR

84. In blood of the man 26 years old 18 % erythrocytes of spherical, flat, domelike and spinous
shape are revealed. Other erythrocytes have the shape of biconcave disks. How such
phenomenon refers to as?
А. *Physiological poikilocytosis
B. Pathological poikilocytosis
С. Physiological anisocytosis
D. Pathological anisocytosis
Е. Erythrocytosis

85. In blood of the patient 14,5 % erythrocytes by a diameter more 8 µm are revealed, 15,5 %
erythrocytes are less than 6 µm, others erythrocytes had a diameter 7,1 - 7,9 µm. How such
phenomenon refers to as?
А. Physiological anisocytosis
B. *Pathological anisocytosis
С. Physiological poikilocytosis
D. Pathological poikilocytosis
Е. Erythrocytosis

86. At the patient blood for the analysis is taken. Its data show, that 35 % erythrocytes have the
irregular form. How such condition refers to as?
А. *Pathological poikilocytosis
B. Anisocytosis
С. Physiological poikilocytosis
D. Macrocytosis
Е. Microcytosis

87. The analysis of blood of the patient has shown acute decrease of the hemoglobin contents.
Which function of blood thus is broken?
А. *Respiratory.
B. Transport.
С. Homeostatic.
D. Protective.
Е. Trophic.

88. At damage of blood vessels there is a spontaneous stop of a bleeding. Name formed element
of blood, which first of all takes part in blood clotting?
А. *Platelets
В. Leukocytes
С. Erythrocytes
D. Lymphocytes
Е. Neutrophils

89. At the analysis of blood of the patient the microcytic anemia was revealed - significant
prevalence of microcytes above normocytes. Which is average diameter of normocytes?
A. 5,1-5,5 µm
B. 6,9-8,1µm
С. *7,1-7,9 µm
D. 8,0-8,9 µm
Е. 4,5-5,5 µm

92. Blood sampling for bulk analysis is recommended to be performed on an empty stomack and
in the morning. What changes in blood composition can occur if to perform blood sampling after
food intake?
A *Increased contents of leukocytes
B Increased contents of erythrocytes
C Increased plasma proteins
D Reduced contents of thrombocytes
E Reduced contents of erythrocytes

93. During the heterotrasplantation of organ it was observed a rejection of graft. What cells of
blood do provide this process?
A *T-lymphocytes-killers
B T- lymphocytes -helpers
C T- lymphocytes -suppressor
D T- lymphocytes-O
E T-lymphocytes-memory

94. Blood taken from a patient for analysis reveals that 30% of red blood cells have irregular
shape. Indicate the name of this phenomenon?
A * Pathological poikilocytosis
B Anisocytosis
C Physiological poikilocytosis
D Macrocytosis
E Microcytosis

95. In a blood smear a large cell with pale basophilic cytoplasm and bean-shaped nucleus was
found. This cell is the biggest among all others in the visible area. What cell is it?
A*Monocyte
B Macrophage
C Plasmocytes
D Middle lymphocyte
E Small lymphocyte

96. In a blood smear, among leucocytes rounded cells with segmented nuclei predominate. The
small granules in their cytoplasm are stained with basophilic due. How are these cells named?
A * Neutrophil
B Basophil
C Eosinophil
D Young neutrophil
E Monocyte

97. In an experiment a population of blood cells were selectively stimulated. As a result, the
permeability of blood vessels increased, and perivascular edema and deceleration of blood
coagulation was observed. What cells of blood were stimulated?
A * Basophils
B Red blood cells
C Platelets
D Eosinophils
E Lymphocytes

98. During investigation of blood smear of a patient some cells were found, which comprise
0,5% of total number of leucocytes; they have S-shaped nuclei and metachromatically stained
granules in their cytoplasm. Name these cells.
A * Basophils
B Neutrophil
C Eosinophil
D Monocyte
E Lymphocytes

99. During histochemical investigation of leucocytes of blood smear, cells with heparin and
histamine in their granules were found. What cells are these?
A * Basophils
B Neutrophils
C Eosinophils
D Monocytes
E Red blood cells

100. During the heterotrasplantation of organs it was revealed the rejection of graft. What cells
mainly take part in this process?
A * T-killers
B Macrophages
C B-lymphocytes
D T-helpers
E T-suppressors

101. In a child, around a scratch on the skin the signs of inflammation were noticed: pain,
redness (rubor), edema as signs of immediate hypersensitiveness. What blood cells are caused
these changes?
A * Basophils
B Eosinophils
C Neutrophil
D Lymphocytes
E Monocytes

102. On specimen of blood smear a cell with segmented nucleus was found. The cytoplasm
contains small granules which are stained with basic as well as with acidic dyes. What is the
name of this cell?
A *Neutrophil
B Eosinophil
C Basophil
D Lymphocytes
E Monocytes

103. In a child (10 years) it was found a helminthosis. What population of leukocytes is expected
to change?
A * Increasing of the number of eosinophils
B Increasing of the number of platelets
C Increasing of the number of red blood cells
D Increasing of the number of neutrophils
E Increasing of the number of basophils

104. During an inspection of patient in a clinic it was revealed acute decrease of hemoglobin.
What function of blood is violated?
A *Respiratory
B Humoral
C Homoeostatic
D Protective
E Trophic

105. The patient addressed to a doctor with complaints on a "running nose", which increases in
spring, in a period flowering of plants. The diagnosis of allergic rhinitis was set. Changes in
leukocyte of which population could be expected?
A *Eosinophilia
B Shifting of formula to the left
C Lymphopenia
D Eosinopenia
E Lymphocytosis

106. During investigating of a blood smear the doctor made a conclusion that blood belongs to a
woman. Features of structure of what blood elements enable to do such conclusion?
A *Neutrophil leucocytes
B Red blood cells
C Lymphocytes
D Monocytes
E Basophils

107. Specimens of two smears were given to the student. On the first - all area of vision is
covered by red blood cells, on the second - blood cells on different stages of development are
determined. What was on specimens?
A * Blood and red bone marrow
B Blood and lymph
C Blood of frog and human blood
D Blood and smear of yellow bone marrow
E Smear of yellow and red bone marrow

108. As a result of investigating of blood spot in place were the crime took place, medico-legal
expert defined that it was blood of a woman. After what signs?
A *Presence of nuclear satellites in neutrophils
B Presence of microcytes and macrocytes
C Phenomena of poykilocytosis
D Presence of specific granules is in eosinophils
E After the amount of red blood cells.

109. In blood of girl of 16 years, who suffers autoimmune inflammation of thyroid, it was found
numerous plasmocytes. With proliferation and differentiation of which cells of blood?
A *B-lymphocytes
B T-helpers
C Tissue basophils
D T-killers
E T-suppressor

110. An ambustial wound was closed by the skin of pig (heterotrasplantation). Indicate effector
cells which will reject the graft (skin of pig).
A *T-killers
B T-helpers
C T-suppressor
D B- lymphocytes
E Natural killers

111. At the second contact of antigen with macro-organism, antibodies are produced by the latter.
To the function of what immunocompetent cells is this phenomenon related?
A * Memory B-lymphocytes
B T-killers
C T-suppressors
D Macrophages
E Dendritic cells

Punctate hemorrhage was found out in the patient after application of a tourniquet. With
dysfunction of what blood cells is it connected?
A * Platelets
B Eosinophils
C Monocytes
D Lymphocytes
E Neutrophils

At the laboratory experiment the leukocyte culture was mixed with staphylococci. Neutrophil
leukocytes engulfed and digested bacterial cells. This processes are termed:
A * Phagocytosis
B Pinocytosis
C Diffusion
D Facilitated diffusion
E Osmosis

A 16-year-old boy was performed an appendectomy. He has been hospitalized for right lower
quadrant abdominal pain within 18 hours. The surgical specimen is edematous and erythematous.
Infiltration by what of the following cells is the most typical for the process occurring here?
A *Neutrophils
B Eosinophils
C Basophils
D Lymphocytes
E Monocytes

A 38-year-old woman was admitted to the admission-diagnostic department with uterine


bleeding. What are the most likely changes of blood?
A *Reduction of hematocrit rate
B Increase of hematocrit rate
C Leukopenia
D Leukocytosis
E Polycythemia

Two hours after an exam a student had a blood count done and it was revealed that he had
leukocytosis without significant leukogram modifications. What is the most probable mechanism
of leukocytosis development?
A *Redistribution of leukocytes in the organism
B Leukopoiesis intensification
C Deceleration of leukocyte lysis
D Deceleration of leukocyte migration to the tissues
E Leukopoiesis intensification

A 38-year-old patient with a uterine hemorrhage lasting for 2 days was delivered to the
admission ward. Which of the following will be revealed in the patient’s blood?
A *Decrease in the hematocrit index
B Eosinophilia
C Deceleration in ESR
D Leukocytosis
E Increase in the color index

A 5-year-old child is ill with measles. Blood analysis revealed increase of total number of
leukocytes up to 13 • 109/l. Leukogram: basophils - 0, eosinophils - 1, myelocytes - 0, juvenile
neutrophils - 0, band neutrophils - 2, segmented neutrophils - 41, lymphocytes - 28, monocytes -
28. Name this phenomenon:
A *Monocytosis
B Agranulocytosis
C Lymphocytosis
D Eosinopenia
E Neutropenia

A patient with clinical presentations of immunodeficiency went through immunological


examinations. They revealed significant loss of cells that form rosettes with erythrocytes of a
ram. What conclusion can be made according to the analysis data?
A * Decrease of T-lymphocytes rate
B Decrease of B-lymphocytes rate
C Decrease of natural killer cell rate
D Decrease of complement system rate
E Insufficiency of effector cells of humoral immunity

After implantation of a cardiac valve a young man constantly takes indirect anticoagulants. His
state was complicated by hemorrhage. What substance content has decreased in blood?
A *Prothrombin
B Haptoglobin
C Heparin
D Creatin
E Ceruloplasmin

The cellular composition of exudate largely depends on the etiological factor of inflammation.
What leukocytes are the first to get into the focus of inflammation caused by pyogenic bacteria?
A *Neutrophil granulocytes
B Monocytes
C Myelocytes
D Eosinophilic granulocytes
E Basophils

In allergic diseases, a dramatic increase in basophilic leukocyte number in patients’ blood is


observed. This phenomenon is due to the following basophil function:
A *Participation of heparin and histamine in metabolism
B Phagocytosis of microorganisms and small particles
C Immunoglobulin synthesis
D Phagocytosis of immune complexes
E Participation in blood clotting

A 3-year-old child has eaten some strawberries. Soon he developed a rash and itching. What was
found in the child’s leukogram?
A *Eosinophilia
B Hypolymphemia
C Neutrophilic leukocytosis
D Monocytosis
E Lymphocytosis

Along with normal hemoglobin types there can be pathological ones in the organism of an adult.
Name one of them:
A *HbS
B HbF
C HbA1
D HbA2
E HbO2

A patient with clinical signs of immunodeficiency has unchanged number and functional activity
of T and B lymphocytes. Dysfunction’s defect of antigen-presentation to the immunocompetent
cells was found during investigation on the molecule level. Defect of what cells is the most
probable here?
A *Macrophages, monocytes
B Т-lymphocytes, В-lymphocytes
C NK-cells
D Fibroblasts, Т-lymphocytes, В- lymphocytes
E 0-lymphocytes

Blood analysis of a patient showed signs of HIV infection (human immunodeficiency virus).
Which cells does HIV-virus primarily affect?
A * Cells that contain receptor T4 (T-helpers)
B Cells that contain receptor IgM(B-lymphocytes)
C Specialized nervous cells (neurons)
D Mast cells
E Proliferating cells (stem hematoplastic cells)

Hemopoiesis. Red bone marrow

205. An electronic microphotograph shows a macrophagic cell with erythrocytes at different


stages of differentiation located along its processes. This is the cell of the following organ:
A *Red bone marrow
B Thymus
C Spleen
D Tonsil
E Lymph node

206. During postembryonal haemopoiesis in the red bone marrow the cells of one of the cellular
differons demonstrate a gradual decrease in cytoplasmic basophilia as well as an increase in
oxyphilia, the nucleus is being forced out. Such morphological changes are typical for the
following haemopoiesis type:
A *Erythropoiesis
B Lymphopoiesis
C Neutrophil cytopoiesis
D Eosinophil cytopoiesis
E Basophil cytopoiesis

228. In punctate of myeloid tissue of child it was revealed cells, during development of which
there is a picnosis and expulsion of nuclei. Indicate the name of hematopoiesis with such
morphological features.
A *Erythropoiesis
B Trombocytopoiesis
C Granulocytopoiesis
D Lymphocytopoiesis
E Monocytopoiesis

229. It is known that in peripheral blood of man megakaryocytes could appear. At what time or
age could these cells be seen in bloodstream?
A *Embryonic period
B Under 1 year
C In age from 1 to 30 years
D In old age
E During pregnancy

230. On an electronic photomicrograph presented the cell of macrophage nature along the
processes of which red blood cells on the different stages of differentiation are located. Which
organ does this cell belong to?
A *Red bone marrow
B Thymus
C Spleen
D Tonsil
E Lymphatic node

231. During histological research of biopsy material of red bone marrow it was revealed
granulocytes. Indicate what changes do occur with the nuclei during differentiation of these cells.
A*Segmentation
B Polyploidisation
C Picnosis
D Enucleation
E Enlargement

232. On the electronic photomicrograph of red bone marrow it was determined megakaryocytes
in peripheral part of cytoplasm of which demarcation ducts were situated. What is the role of
these structures?
A *Formation of platelets
B Increase of surface area of the cell
C Increase of amount of the ionic channels
D Reproduction of the cell
E Destruction of cell

233. At the inspection of patient of 35 years the histological investigation of red bone marrow
punctate was done. A considerable increase in quantity of megakaryocytes was revealed. What
changes of peripheral blood is it accompanied with?
A * Augmentation of quantity of platelets
B Augmentation of quantity of leucocytes
C Diminishing of quantity of platelets
D Diminishing of quantity of granulocytes
E Diminishing of quantity of leucocytes

Blood count of an athlete is as follows: erythrocytes - 5, 5 • 1012/l, Hb- 180 g/l, leukocytes - 7 •
109/l, neutrophils - 64%, basophils - 0,5%, eosinophils - 0,5%, monocytes - 8%, lymphocytes -
27%. First of all, such results indicate the stimulation of:
A * Erythropoiesis
B Leukopoiesis
C Lymphopoiesis
D Granulocytopoiesis
E Immunogenesis

In which of the following cells involved in erythropoiesis does hemoglobin synthesis begin?
a. Orthochromatic erythroblast
b. Polychromatophilic erythroblast
c. Reticulocyte
d. *Basophilic erythroblast
e. Proerythroblast

Which of the following can be used to describe megakaryocytes?


a. Multinucleated
b. Formed by fusion of haploid cells
c. Precursors to bone marrow macrophages
d. A minor but normal formed element found in the circulation
e. *Possess dynamic cell projections from which one type of formed element is released

Which cytoplasmic components are the main constituents of the dark precipitate that forms in
reticulocytes upon staining with the dye cresyl blue?
a. Golgi complexes
b. Hemoglobin
c. Nucleoli
d. Nuclear fragments
e. *Polyribosomes

Which process occurs during granulopoiesis but not during erythropoiesis?


a. Cells lose their capacity for mitosis
b. Euchromatin content increases
c. *Nucleus becomes increasingly lobulated
d. Overall cell diameter decreases
e. Overall nuclear diameter decreases
What fate often awaits granulocytes that have entered the margination compartment?
a. Undergo mitosis
b. *Crossing the wall of a venule to enter connective tissue
c. Cannot reenter the circulation
d. Differentiate into functional macrophages
e. Begin to release platelets

What is the earliest stage at which specific granulocyte types can be distinguished from one
another?
a. *Myelocyte
b. Band form
c. Reticulocyte
d. Metamyelocyte
e. Promyelocyte

Which cell type is capable of further mitosis after leaving the hemopoietic organ in which it is
formed?
a. Basophil
b. Eosinophil
c. Reticulocyte
d. *Lymphocyte
e. Neutrophil

Shortly after her birth a baby is diagnosed with a mutation in the erythropoietin receptor gene
which leads to familial erythrocytosis (familial polycythemia). During the seventh to ninth
months of fetal development, the primary effect on her red blood cell production was in which of
the following?
a. Liver
b. Yolk sac
c. Spleen
d. Thymus
e. *Bone marrow

A 54-year-old man presents with recurrent breathlessness and chronic fatigue. After routine tests
followed by a bone marrow biopsy he is diagnosed with lymphocytic leukemia. Chemotherapy is
administered to remove the cancerous cells, which also destroys the precursor cells of
erythrocytes. To reestablish the erythrocytic lineage, which of the following cells should be
transplanted?
a. Reticulocytes
b. Orthochromatophilic erythroblasts
c. Megakaryoblasts
d. *Basophilic erythroblasts
e. Metamyelocytes

A smear of blood from a 70-year-old leukemia patient reveals a larger than normal population of
cells that have large, round nuclei with 1 or 2 nucleoli. The cytoplasm of these cells shows
azurophilic granules. Which of the following forms of leukemia would you suspect?
a. *Promyelocytic leukemia
b. Basophilic leukemia
c. Lymphoblastic leukemia
d. Stem cell leukemia
e. Eosinophilic leukemia
4. Which of the following is derived from CFU-E?
A *Red blood cell
B Lymphocyte
C Platelet
D Neutrophil
E Basophil

Lymphoid organs

200. A histological specimen presents an organ that has both cortical and medullary substance.
Cortical substance consists of an external zone that contains lymph nodules as well as of a
paracortical zone. Medullary substance contains medullary cords, sinuses and trabecules. What
organ possesses these morphological signs?
A *Lymph node
B Spleen
C Kidney
D Thymus
E Adrenal glands

201. In a histological specimen parenchyma of an organ is represented by lymphoid tissue that


forms lymph nodes; the latter are arranged in a diffuse manner and enclose a central artery. What
anatomic formation has such morphological structure?
A *Spleen
B Tonsil
C Lymph node
D Thymus
E Red bone marrow

202. Medullary substance of a hemopoietic organ's lobule in a histological specimen is lighter


colored and contains epithelial bodies. What organ are these morphological properties typical
for?
A *Thymus
B Lymph node
C Spleen
D Liver
E Kidney

204. A specimen shows an organ covered with the connective tissue capsule with trabeculae
radiating inward the organ. There is also cortex containing some lymph nodules, and medullary
cords made of lymphoid cells. What organ is under study?
A *Lymph node
B Thymus
C Spleen
D Red bone marrow
E Tonsils

207. On a histological specimen parenchyma of an organ is represented by lymphoid tissue that


forms lymph nodes; the latter are arranged in a diffuse manner and enclose a central artery. What
anatomic formation has such morphological structure?
A *Spleen
B Tonsil
C Lymph node
D Thymus
E Red bone marrow

208. The specimens present sections of haemopoetic and immunogenetic organs. Organ has
lymph tissue forming different structures (lymph nodes,lobules, bars). In what organ does
antigen-independent proliferation and differentiation take place?
A *Thymus
B Lymphatic nodes
C Spleen
D Hemolymph nodes
E Tonsil

210. On specimen, there is an organ covered by connective tissue capsule, which send trabecules
to the parenchyma. In this organ, it is possible to distinguish a cortex where lymphatic nodules
are located and medulla, represented by medullary cords made up of lymphoid cells. What organ
is presented on specimen?
A * Lymphatic node
B Thymus
C Spleen
D Red bone marrow
E Tonsil

211. On histological specimen presented the organ of hematopoiesis and immune defense, which
is built from lobules which are surrounded by the layers of connective tissue, on periphery of
lobules the amount of cells is considerably more high than in a center, lymphatic nodules are
absent. What organ is it?
A * Thymus
B Red bone marrow
C Lymphatic node
D Spleen
E Tonsil

212. At histological investigation of thymus of man by age 40 years, it was revealed that there
was a decrease in the number of parenchymal elements of thymus, increasing of adipose and
connective tissue, high quantity of thymic corpuscles (Hassall's corpuscles), at normal total mass
of organ. How is such phenomenon called?
A *Age involution of thymus
B Accidental involution of thymus
C Hypotrophy of thymus
D Dystrophy of thymus
E Atrophy of thymus

214. Child has immunodeficiency. Cellular immunity suffers and it results in frequent viral
infections. By violation in function of what organ is it caused?
A *Thymus
B Red bone marrow
C Lymphatic nodes
D Spleen
E Palatal tonsil

215. Two histological specimens are given to a student. On both specimens there are organs
which have lymphatic nodules. On first specimen - only follicles, and on the second - follicles
with eccentrically located artery. Define what organs are these?
A * First - lymphatic node, second - spleen
B First - red bone marrow, second - spleen C First - thymus, second - spleen
D First - liver, second - lymphatic node
E First - liver, second - spleen
216. On a specimen, a spherical elements made up of lymphocytes can be seen. In a middle of
this elements there is a central artery. What organ is this?
A* Spleen
B Kidney
C Thymus
D Bone marrow
E Lymphatic node

217. On a specimen with the contours of bean-shape organ there is a cortex and medulla. A
cortex is represented by separate spherical nodes 0,5 .1 mm in diameter, and medulla - by
medullary cords. What organ is it?
A * Lymphatic node
B Kidney
C Thymus
D Adrenal gland
E Spleen

218. On the specimen of small intestine, in the lamina propria of mucosa it was revealed an
accumulation of cells of spherical shape with large basophilic nuclei, which were surrounded by
narrow rim of cytoplasm. In majority of such accumulations central part was lighter and
contained fewer cells, than peripheral. What morphological structure do such accumulations
belong to?
A * Lymphatic node
B Nervous ganglion
C Adipose cells
D Blood vessels
E Lymphatic vessels

219. Medulla of lobule of hematopoietic organ on histological specimen has more light color and
contains little epithelial corpuscles. What organ do these morphological signs belong to?
A *Thymus
B Lymphatic node
C Spleen
D Liver
E Kidney

220. On histological specimen presented an organ in which lymphocytes form three kinds of
lymphoid structures: lymphatic nodules, medullary cords and sinuses. What organ is it?
A *Lymphatic node
B Spleen
C Thymus
D Tonsil
E Red bone marrow

222. On specimen presented the organ, in its reticular stroma there were located mature blood
elements and lymphoid elements. What organ is presented on specimen?
A *Spleen
B Lymphatic node
C Tonsil
D Thymus
E Red bone marrow

223. On an electronic photomicrograph presented the cell, with protrusions and deep
invaginations in cytolemma, in which lymphocytes are differentiated. What organ is it?
A *Thymus
B Red bone marrow
C Spleen
D Tonsil
E Liver

224. A microsection is done through a lymphatic node. On a slice there is an extension of its
paracortical area. Proliferation of what type of cells of lymphatic node did causes this process?
A * T-cell
B Dendritic cells
C Plasmocytes
D Macrophages
E Reticulocytes

225. At a child with the broken immune reactivity it was performed a study of antigen-
independent proliferation and differentiation of T-cells. Punctate of what organ was taken for
research?
A *Thymus
B Spleen
C Lymphatic node
D Red bone marrow
E Palatal tonsil

226. A new-born child has an excalation of thymus. What type of hemopoiesis will be broken?
A *Lymphopoiesis
B Monocytopoiesis
C Erythropoiesis
D Granulocytopoiesis
E Megakariocytopoiesis

227. At the patient it is observed enlargement of spleen and decreasing of amount of erythrocytes
in peripheral blood. Due to hyper function of what type of cells in the spleen this phenomenon
could be explained?
A *Macrophages
B Lymphocytes
C Dendritic cells
D Plasmocytes
E Reticulocytes

A teenager was irradiated with high radiation dose that resulted in serious damages of lymphoid
system, lysis of many lymphocytes. Restoration of normal hemogram is possible due to the
functioning of the following gland:
A *Thymus
B Thyroid
C Liver
D Pancreas
E Adrenal

During autopsy of a 9-month-old girl’s body, who died due to severe pneumonia complicated
with sepsis, lack of thymus is observed. In the lymph nodes the lymphoid follicles and cortical
substance are absent; follicles of spleen are reduced in size with 20 no light zones and plasma
cells. What is the cause of such structural changes?
A *Thymus agenesis
B Accidental involution of thymus
C Thymus hypoplasia
D Thymus atrophy
E Thymus aplasia
Histological specimen of a hemopoietic organ shows clusters of node- and band shaped
lymphocytes that along with stroma elements compose cortical and medullar substances. Name
this organ:
A *Lymph node
B Spleen
C Red bone marrow
D Thymus
E Palatine tonsil

A doctor examined a patient, studied the blood analyses, and reached a conclusion, that
peripheral immunogenesis organs are affected. What organs are the most likely to be affected?
A *Tonsils
B Thymus
C Kidneys
D Red bone marrow
E Yellow bone marrow

Histological specimen shows organ parenchyma to consist of lymphoid tissue that forms lymph
nodules; the nodules are located diffusely and have a central artery. What anatomical structure
has such morphological characteristics?
A. *Spleen
B. Tonsil
C. Lymph node
D. Thymus
E. Red bone marrow

Histological specimen shows organ parenchyma to consist of lymphoid tissue that forms lymph
nodules; the nodules are located diffusely and have acentral artery. What anatomical structure
has such morphological characteristics?
A. *Spleen
B. Tonsil
C. Lymph node
D. Thymus
E. Red bone marrow

Connective tissues

112. A specimen of connective tissue of derma was stained with Sudan III and hematoxylin.
There are clusters of big polygonal cells that turned orange. Their nuclei are flattened and located
on periphery. What tissue is it?
A *White adipose
B Brown adipose
C Reticular connective
D Hyaline cartilaginous
E Lamellar osseous

113. Which of the following is NOT primarily composed of connective tissue?


A. Bone marrow
B. Articular cartilage
C. *Heart
D. Mesenchyme
E. Fat

114. Which one of these cells is not a cell type routinely found in loose connective tissue?
A. Fibroblast
B. *Microglia
C. Histiocyte
D. Plasma cell
E. Mast cell

115.Which connective tissue cell is a tissue macrophage?


A. Fibroblast
B. Myofibroblast
C. *Histiocyte
D. Plasma cell
E. Mast cell

116. Which of the following can be classified as "specialized connective tissue"?


A. Cartilage
B. Loose connective tissue
C. Mesenchyme
D. Dense connective tissue
E. *Mucous connective tissue

117. Which of the following can be classified as "connective tissue proper"?


A. Adipose tissue
B. *dense irregular connective tissue
C. Bone
D. Blood
E. Cartilage

118. What type of tissue is Wharton's jelly?


A. *mucous connective tissue
B. Mesenchyme
C. Loose irregular connective tissue
D. Dense irregular connective tissue
E. Dense regular connective tissue

119. What type of tissue is a tendon composed of?


A. Mucous connective tissue
B. Mesenchyme
C. Loose irregular connective tissue
D. Dense irregular connective tissue
E. *Dense regular connective tissue

120. What does connective tissue develop from?


A. Mesothelium
B. *Mesenchyme
C. Mesangial cells
D. Mesentery
E. Wharton's jelly

121. Which of the following is a component of the ground substance?


A. Hyaluronic acid
B. Proteoglycans
C. Glycosaminoglycans
D. Chondroitin sulfate
E. *All of the above

122. Decreased blood supply to the organs causes hypoxia that activates fibroblasts function.
Volume of what elements is increased in this case?
A *Intercellular substance
B Vessels of microcircular stream
C Nerve elements
D Parenchymatous elements of the organ
E Lymphatic vessels

123. Live vaccine is injected into the human body. Increasing activity of what cells of connective
tissue can be expected?
A *Plasmocytes and lymphocytes
B Macrophages and fibroblasts
C Pigmentocytes and pericytes
D Adipocytes and adventitious cells
E Fibroblasts and labrocytes

124. In course of an experiment a big number of stem cells of red bone marrow was in some way
destructed. Regeneration of which cell populations in the loose connective tissue will be
inhibited?
A *Of macrophages
B Of fibroblasts
C Of pigment cells
D Of lipocytes
E Of pericytes

126. During development of clinical displays of allergy a leading role is played by histamine.
What cells do produce this substance?
A * Mast cells
B T- lymphocytes
C Macrophages
D B- lymphocytes
E Plasmocytes

127. In an experiment B- lymphocytes of the blood were labeled by marker. The foreign protein
was injected under a skin of experimental animal. What cells of connective tissue will include
this marker?
A * Plasmocytes
B T- lymphocytes
C Macrophages
D Tissue basophils
E Fibroblasts

128. During wound healing the connective tissue scar develops in the area of tissue defect. What
cells provide this process?
A * Fibroblasts
B Macrophages
C Fibrocytes
D Tissue basophils
E Melanocytes

129. During training a sportsman's lower extremity was traumatized. Traumatologist set a
diagnosis: tendon rupture. To which type of connective tissue does a tendon belong?
A *Dense regular connective tissue
B Dense irregular connective tissue
C Loose connective tissue
D Reticular tissue
E Cartilaginous tissue

130. With age there are skin changes which results in decreasing of it's elasticity. What elements
of connective tissue do provide this property?
A * Collagen and elastic fibers
B Ground substance
C Cells of epidermis
D Cells of connective tissue
E Reticular fibers

133. After chemical burn of esophagus, local narrowing of its lumen developed as a result of scar
formation. What cells of loose connective tissue do take part in formation of scars?
A * Mature specialized fibroblasts
B Young non-specialized fibroblasts
C Fibrocytes
D Myofibroblasts
E Fibroclasts

134. As a result of thrombosis of left coronary artery death of group of cardiomyocytes occurred
(infarction of myocardium). What cells will provide reparative regeneration in the area of
damage?
A *Fibroblasts
B Neighboring cardiomyocytes
C Myosimplast
D Myosatellites
E Smooth muscle cells

137. During the observation of histological specimen of lymph node from an experimental
animal after antigen stimulation in the medullary cords a lot of cells were found of following
morphology: intensively basophilic cytoplasm, eccentrically placed nucleus with a chromatin,
located as "cart-wheel" and light area of cytoplasm around nucleus. What cells is it?
A *Plasmocytes
B Macrophages
C Fibroblasts
D Adipocytes
E Tissue basophils (mast cells)

139. During histological specimen observation of connective tissue it was found large cells,
filled with basophilic metachromatic granules; with the help of histochemistry it was revealed,
that granules contain heparin and histamine. What cells were most probably found in specimen?
A *Mast cells
B Fibroblasts
C Macrophages
D Plasmocytes
E Adipocytes

140. During an experiment a lot of stem cells of red bone marrow were destroyed. A renewal of
what populations of cells of connective tissue will be violated?
A * Macrophages
B Fibroblasts
C Pigment cells
D Lipocytes
E Pericytes

142. To the experimental animal it was injected a media, which violates formation of collagen
fibers. How will it affect properties of tendon?
A * Tensile strength will decrease
B Will not change
C Elasticity will decrease
D Elasticity and tensile strength will decrease
E Strength will increase, elasticity will decrease
Decreased blood supply to the organs causes hypoxia that activates fibroblasts function. Volume
of what elements is increased in this case?
A Intercellular substance
B Vessels of microcircular stream
C Nerve elements
D Parenchymatous elements of the organ
E Lymphatic vessels

In course of histidine catabolism a biogenic amine is formed that has powerful vasodilatating
effect. Name it:
A Histamine
B Serotonin
C Dioxyphenylalanine
D Noradrenalin
E Dopamine

A microslide presents a tissue with spherical cells, each of them containing a large fat drop
covered with thin cytoplasm layer in its center. Nucleus is compressed and situated at the cell
periphery. What tissue is it?
A White adipose tissue
B Brown adipose tissue
C Mucous tissue
D Pigmented tissue
E Reticular tissue

Skeletal tissue

144. Calcification of the intercellular substance of bone tissue is accompanied by the deposition
of hydroxyapatite crystals along the collagen fibers. This process requires the presence of
alkaline phosphatase in the intercellular substance. What cell produces this enzyme?
A Osteoblast
B Osteocyte
C Osteoclast
D Chondroblast
E Chondrocyte

146. A histological specimen presents the tissue that contains cells having no processes and a few
tens of nuclei each. One of cell surfaces has a corrugated zone that provides secretion of
hydrolytic elements. What tissue is it?
A *Bone tissue
B Cartilaginous tissue
C Epithelial tissue
D Nerve tissue
E Muscular tissue

147. Examination of a histological specimen of tubular bone revealed signs of regeneration


process (callus). What tissue is this structured formed of?
A * Woven bone
B Loose connective
C Reticular
D Epithelial
E Lamellar osseous

148. The symptoms of regeneration process (callus) on the place of fracture were revealed in the
histologic specimen of tubular bone. What tissue forms this structure?
A * Woven bone tissue
B Loose connective tissue
C Reticular tissue
D Epithelial tissue
E Lamellar bone tissue

149. What cell produces the cartilaginous matrix?


A. Chondrocyte
B. *Chondroblast
C. Osteocyte
D. Osteoclast
E. Bone lining cell

150. Which type of cartilage is found in the larynx?


A. Hyaline cartilage
B. Elastic cartilage
C. Fibrocartilage
D. *Both A and B
E. All of the above

151. Which of the following is NOT a glycosaminoglycan in cartilage?


A. Chondroitin sulfate
B. *Proteoglycans
C. Keratan sulfate
D. Hyaluronic acid
E. All of the above are glycosaminoglycans in cartilage

152. Which type of cartilage is characterized by a glassy matrix?


A. *Hyaline cartilage
B. Elastic cartilage
C. Fibrocartilage
D. All of the above
E. None of the above

153. Which type of cartilage is characterized by the presence of chondrocytes sitting in lacunae?
A. Hyaline cartilage
B. Elastic cartilage
C. Fibrocartilage
D. *All of the above
E. None of the above

154. Which type of cartilage is the most abundant?


A. *Hyaline cartilage
B. Elastic cartilage
C. Fibrocartilage
D. Hyaline cartilage and elastic cartilage equally
E. Elastic cartilage and fibrocartilage equally

155. Which type of cartilage forms the articular surface on bones?


A. *Hyaline cartilage
B. Elastic cartilage
C. Fibrocartilage
D. All of the above
E. None of the above

156. Which type of cartilage is found in the external ear?


A. Hyaline cartilage
B. *Elastic cartilage
C. Fibrocartilage
D. All of the above
E. None of the above

157. Costal cartilage is composed of what type of cartilage?


A. *Hyaline cartilage
B. Elastic cartilage
C. Fibrocartilage
D. All of the above
E. None of the above

158. Which type of cartilage forms the symphysis pubis?


A. Hyaline cartilage
B. Elastic cartilage
C. *Fibrocartilage
D. All of the above
E. None of the above

159. What is dense bone?


A. Immature bone
B. Cancellous bone
C. *Compact bone
D. Woven bone
E. Spongy bone

160. Which cell is a resting osteoblast?


A. Chondrocyte
B. Chondroblast
C. Osteocyte
D. Osteoclast
E. *Bone lining cell

161. What are the mineral crystals in bone called?


A. *Hydroxyapatite
B. Calcite
C. Tourmaline
D. Rubellite
E. Indicolite

162. What is the cylindrical structure in compact bone?


A. Osteoclast
B. *Osteon
C. Osteocyte
D. Osteoblast
E. Osteoid

163. What are Sharpey's fibers?


A. Elastic fibers
B. *Collagen fibers
C. Reticular fibers
D. Trabeculae
E. Dense regular connective tissue

164. What is the space that an osteocyte rests in?


A. Canaliculi
B. Sharpey's fibers
C. Trabeculae
D. Tome's process
E. *Lacuna

165. What is bone formation called when the bone is formed from a cartilage template?
A. Intraosseous
B. En bloc
C. Intramembranous
D. *Endochondral
E. Endosteum

166. What is the primary component of red marrow?


A. *Hematopoietic tissue
B. Fat
C. Cartilage
D. Fibrous tissue
E. Bone

167. What cell is an immature bone cell?


A. Osteoclast
B. Osteon
C. Osteocyte
D. *Osteoblast
E. Osteoid

168. What is bundle bone?


A. Cancellous bone
B. Compact bone
C. Dense bone
D. Spongy bone
E. *Immature bone

169. As a result of a chest trauma the costal cartilage was damaged. The cartilage regenerates due
to the following layer of perichondrium:
A *Chondrogenic
B Fibrous
C Elastic
D Collagen
E Sharpey's fibers

A connective tissue specimen stained with hematoxylin-eosin shows isogenous cell groups
surrounded with basophilic intercellular substance. No fibrous structures are detected. What type
of connective tissue is it?
A *Hyaline cartilage tissue
B Elastic cartilage tissue
C Dense fibrous tissue
D Loose fibrous tissue
E Splenial bone tissue

X-ray examination of a 57-year-old man indicates local areas of hard bone tissue resorption in
some of the patient’s bones. These changes can be associated with increased activity of:
A. *Osteoclasts
B. Chondroblasts
C. Osteocytes
D. Osteoblasts
E. Chondrocytes

125. In course of indirect histogenesis of tubular bone tissue a plate is formed between
epiphyseal and diaphyseal ossification centers that provides further lengthwise growth of bones.
What structure is it?
A *Metaphyseal plate
B Osseous cuff
C Osseous plate
D Osteon
E Layer of interior general plates

131. Elderly people have an excessive loss of bone tissue mass, which reflects osteoporosis.
Activating of what cells of bone tissue does cause development of this disease?
A *Osteoclasts
B Osteoblasts
C Macrophages
D Tissue basophils
E Osteocytes

132. In specimen there is a tissue, which contains cells, devoid of processes with several dozens
of nuclei. One of the surfaces of these cells has ruffled border area, through which secretion of
hydrolases occurs. What tissue is presented in this specimen?
A *Bone tissue
B Cartilaginous tissue
C Epithelial tissue
D Nervous tissue
E Muscular tissue

135. In a histological specimen the isogenous groups of cells are observed. What cells are
initial(precusors) in formation of these groups?
A Chondrocytes of I type
B *Chondroblasts
C Prechondroblasts
D Chondrocytes of II type
E Chondrocytes of III type

136. It is known, that cartilages of joints do not have perichondrium. What type of growth do
take place during processes of their regeneration?
A *Interstitial
B Appositional
C By imposition
D Appositional and interstitial
E-

141. During the endochondral ossification of bone tissue of tubular bones a plate appears
between the epiphyseal and diaphyseal centers of ossification, which provides growth of bones
in length. Indicate the name of this structure.
A *Metaepiphyseal plate
B Bone cuff
C Bone plate
D Osteon
E Layer of internal general plates

Muscular tissues

170. The destruction of the thick myofilaments in striated muscle occurs after mechanical
trauma. What structure bounded with myosin filaments is damaged also?
А. *Mesophragma.
В. Mesopfragma and telophragma.
С. Telofragma.
D. Cytolemma.
E. Cytolemma and telophragma.

171. The destruction of the thin myofilaments in striated muscle occurs after action of hydrolytic
enzymes. What structures are damaged?
А. *Actin myofilaments.
В. Myosin myofilaments.
С. Tonofibril.
D. Tropocollagen complexes.
E. Nucleoprotein complexes.

172. The structural unit of tissue is a fiber which consists of symplast and satellitocytes covered
by general basal membrane. Indicate tissue what has such construction.
А. *Skeletal striated muscle.
В. Smooth muscle.
С. Cardiac muscle.
D. Loose connective tissue.
E. Reticular tissue.

173. The atrophy of the muscles what appeared as a result of dysbolism of the proteins is
discovered at patient. Name the protein what participates in formation of the thin myofilaments?
А. *Actin
В. Tubulin
С. Dinein
D. Desmin
E. Keratin

174. The skeletal myofibers are damaged at child 14-years old after trauma. Name the source of
reparative regeneration of the skeleton myofibers?
A. *Myosatellitocytes.
B. Myofibrils.
C. Myofilaments.
D. Sarcolemma.
E. Endomysium.

175. Mesenchyme cells are damaged experimental. The development which tissue will broke?
A. Skeletal muscle.
B. Cardiac muscle.
C. *Smooth muscle.
D. Myoepithelial cells.
E. Muscle of the iris.

176. The fibers containing numerous nucleuses are seen in preparation of muscle tissue. The
nucleuses are situated peripheral. What type of tissue is presented in preparation?
A. *Skeletal muscle.
B. Cardiac muscle.
C. Smooth muscle.
D. Myoepithelial cells.
E. Muscle of the iris.

177. A microspecimen of the submandibular salivary gland shows some basket-shaped cells
concentrated around the acines and excretory ducts. These cells surround bases of the serous
cells and are called myoepitheliocytes. These cells relate to the following tissue:
A *Muscular tissue
B Epithelial tissue
C Neural tissue
D Special connective tissue
E Loose fibrous connective tissue

178. What is the connective tissue covering of a muscle fascicle?


A. Sarcolemma
B. Endomysium
C. Epimysium
D. Sarcoplasm
E. *Perimysium

179. What is actin?


A. *Myofilament
B. Myosin
C. Muscle fibers
D. Myofibrils
E. Myocardium

180. Which of the following is composed of smooth muscle?


A. Upper esophagus
B. Heart
C. Tongue
D. Biceps muscle
E. *Walls of the visceral organs

181. What is a receptor in muscle?


A. Motor unit
B. Motor neuron
C. Motor end plate
D. *Neuromuscular spindle
E. Neurotransmitter

182. Which fiber type is larger in diameter?


A. Red fibers
B. *White fibers
C. Intermediate fibers
D. All of the above
E. None of the above

183. Which fiber type is make up fast-twitch muscle?


A. Red fibers
B. *White fibers
C. Intermediate fibers
D. All of the above
E. None of the above

184. Which fiber type has more myoglobin?


A. *Red fibers
B. White fibers
C. Intermediate fibers
D. All of the above
E. None of the above

185. Which fiber type gets its energy primarily from glycogen?
A. Red fibers
B. *White fibers
C. Intermediate fibers
D. All of the above
E. None of the above
186. Which fiber type is seen in skeletal muscle?
A. Red fibers
B. White fibers
C. Intermediate fibers
D. *All of the above
E. None of the above

189. Patient with injured muscles of the lower extremities was admitted to the traumatological
department. Due to what cells is reparative regeneration of the muscle fibers and restoration of
the muscle function possible?
A *Satellite-cells
B Myoblasts
C Myofibroblasts
D Fibroblasts
E Myoepithelial cells

190. In course of a conditional experiment the development of mesenchymal cells was


completely inhibited. Development of the following muscular tissue will be disturbed:
A *Smooth muscular tissue
B Neural muscular tissue
C Epidermal muscular tissue
D Cardiac muscular tissue
E Skeletal muscular tissue

191. A microspecimen of the submandibular salivary gland shows some basket-shaped cells
concentrated around the acines and excretory ducts. These cells surround bases of the serous
cells and are called myoepitheliocytes. These cells relate to the following tissue:
A *Muscular tissue
B Epithelial tissue
C Neural tissue
D Special connective tissue
E Loose fibrous connective tissue

192. On a histological specimen presented the tissue the main element of which is fiber, which is
composed of myosymplast and satellitocytes, covered by common basal membrane. What tissue
is it?
A *Skeletal striated muscular tissue
B Smooth muscular tissue
C Cardiac muscular tissue
D Loose connective tissue
E Reticular tissue

193. In the phase of myocardium contraction (systole) the concentration of calcium ions is
sharply increased in the sarcoplasm of cardiomyocytes. What structures take part in deposition of
calcium ions?
A * L - system
B Lysosomes
C. Ribosomes
D. T-system
E. Nucleolus

194. On the histological specimen of the heart, it was distinguished cells of rectangular form by
sizes ranging from 50 to 120 µm, with centrally located nuclei, well-developed myofibrils,
coupled together with the help of intercalated disks. Choose the function which is coupled with
these cells:
A* Contraction of heart
B Conducting of impulses
C Endocrine
D Protective
E Regenerative

195. On the electronic photomicrograph of a cell which has a stick-like nucleus and fusiform
shape, there are plenty of intermediate filaments of desmin in the cytoplasm. What tissue is it?
A *Muscular
B Nervous
C Epithelial
D Connective
E-

196. In a conditional experiment, the development of mesenchymal cells was fully inhibited.
Violation of development of what muscular tissue will be observed?
A *Smooth muscular tissue
B Muscular tissue of neural origin
C Muscular tissue of epidermal origin
D Cardiac muscular tissue
E Skeletal muscular tissue

197. At an investigation of striated muscle fiber after the action of hydrolytic enzymes, there is
destruction of thin myofilaments. Which structures were damaged?
A *Actin myofilaments
B Tonofibrils
C T - system
D Sarcoplasmic reticulum
E Myosin filaments

198. On a drawing of the structural unit of myofibril of striated muscles, a sarcomere was
schematically represented. The sarcomere is situated between two nearby Z-lines. How will the
H-area change at maximal contraction of sarcomere?
A * Disappears.
B Does not change
C Increased in two times
D Diminishes in two times
E Occupies all sarcomere

Negative environmental factors have caused the dysfunction of myosatellite cells. What function
of the whole muscle fibre is likely to be changed in this case?
A *Regeneration
B Contraction
C Trophism
D Contractile thermogenesis
E Relaxation

A microspecimen of heart shows rectangular cells from 50 to 120 micrometers large with central
position of nucleus and developed myofibrils. The cells are connected by intercalated discs.
These cells are responsible for the following function:
A *Function of heart contractions
B Function of impulse conduction
C Endocrine
D Protective
E Regeneratory

Nerve tissue

As a result of a mechanical injury an over 10 cm long portion of a peripheral nerve was


damaged. This led to the impairment of the upper limb activity. The patient was offered nerve
transplantation. What glial cells will participate in regeneration and provide the trophism of the
injured limb?
A Schwann cells
B Fibrous cells
C Protoplasmic cells
D Microglia
E Ependymal cells

Which of the following is characteristic of the chromatophilic material called Nissl substance in
neural tissue?
a. Found throughout neurons
b. *Site of mRNA translation for proteins of the axolemma
c. Most abundant in unipolar neurons
d. Becomes more abundant as an individual gets older
e. An example of intermediate filament proteins

Which of the following events occurs immediately after an action potential reaches a synapse at
an axon terminal?
a. Vesicle fusion with the presynaptic terminal membrane
b. *Calcium ion influx at the presynaptic terminal
c. Neurotransmitter binding to receptors on the postsynaptic membrane
d. Neurotransmitter release into the synaptic cleft
e. Binding of the neurotransmitter at the presynaptic terminal

A 35-year-old woman presents with weakness and spasticity in the lower left extremity, visual
impairment and throbbing in the left eye, difficulties with balance, fatigue, and malaise. There is
an increase in cerebrospinal fluid (CSF) protein, elevated gamma globulin, and moderate
pleocytosis. MRI confirms areas of demyelination in the anterior corpus callosum. Imaging
identifies plaques which are hyperintense on T2-weighted and fluid attenuated inversion
recovery (FLAIR) images, and hypointense on T1-weighted scans. Which of the following cells
are specifically targeted in her condition?
a. Microglia
b. *Oligodendrocytes
c. Astrocytes
d. Schwann cells
e. Multipolar neurons

A 22-year-old man receives a severe, traumatic compression injury to his radial nerve during a
motorcycle crash. He shows an advancing Tinel sign. Which one of the following characterizes
regeneration of axons after this nerve injury?
a. It occurs in the absence of motor nerve action potentials.
b. It occurs at a rate of about 100 mm/d.
c. It occurs in the segment distal to the site of axon damage.
d. *It occurs by a process that involves Schwann cell proliferation.
e. It occurs in conjunction with degeneration and phagocytosis of the endoneurium.

236. A sensory nerve ganglion consists of roundish neurocytes with one process that divides into
axon and dendrite at a certain distance from perikaryon. What are such cells called?
A *Pseudounipolar
B Unipolar
C Bipolar
D Multipolar
E Apolar

Which of the following statements regarding nerve cell membrane potentials is true?
A Membrane potentials are maintained at rest by Na + entering the cell.
B Entrance of K+ causes the membrane to return to its resting potential.
C *Depolarization triggers the opening of voltage-gated K+ channels.
D Voltage-gated Na + channels become activated during the refractory period.
E The influx of K+ reverses the resting potential.

Which of the following statements is characteristic of the perineurium?


A It is a fascia surrounding many bundles of nerve fibers.
B It is the fascia surrounding a single nerve fiber.
C It is a thin layer of reticular fibers covering individual nerve fibers.
D It is a fascia that excludes macromolecules and forms the external coat of nerves.
E *It consists in part of epithelioid cells that surround a bundle (fascicle) of nerve fibers.

Which one of the following neurotransmitters functions to increase cardiac output?


A Dopamine
B Serotonin
C *Norepinephrine
D Glutamate
E GABA

Nissl bodies are composed of


A synaptic vesicles and acetylcholine.
B *polyribosomes and rough endoplasmic reticulum.
C lipoprotein and melanin.
D neurofilaments and microtubules.
E SER and mitochondria.

The axon hillock contains


A rough endoplasmic reticulum.
B ribosomes.
C *microtubules.
D Golgi complex.
E synaptic vesicles.

Synaptic vesicles possess which of the following characteristics?


A Manufacture neurotransmitter
B Enter the synaptic cleft
C *Become incorporated into the presynaptic membrane
D Become incorporated into the postsynaptic membrane
E Release neurotransmitter via endocytosis

Myelination of peripheral nerves is accomplished by


A astrocytes.
B oligodendrocytes.
C *Schwann cells.
D neural crest cells.
E basket cells.

Episodes of demyelination are associated with


A meningitis.
B Huntington chorea.
C spina bifida.
D Parkinson disease.
E *multiple sclerosis.

Tremors, shuffling gate, and masklike facial expressions are associated with
A meningitis.
B Huntington chorea.
C spina bifida.
D *Parkinson disease.
E multiple sclerosis.

Loss of neurotransmitter GABA is associated with


A meningitis.
B *Huntington chorea.
C spina bifida.
D Parkinson disease.
E multiple sclerosis.

Deterioration and death of the dopaminergic neurons within the substantia nigra of the brain are
associated with
A meningitis.
B Huntington chorea.
C spina bifida.
D *Parkinson disease.
E multiple sclerosis.

A 25-year-old woman who are suffering from disseminated sclerosis myelination of nerve fibers
is violated. Myelination of peripheral nerves is accomplished by:
A. *Schwann cells
B. Astrocytes
C. Oligodendrocytes
D. Neural crest cells
E. Basket cells

In electron micrograph of a nervous cell with plenty of organelles in a cytoplasm is represented.


What type of cellular structures does the chromatophilic substance of neuron behave to?
A. *Organelles of protein synthesizing apparatus
B. Mitochondria
C. Lysosomes
D. Dictyosome of Golgi complex
E. Cytoskeleton

After the traumatic squeezing of extremity, the degeneration of nervous fibers is found in its
nervous trunk. There are all displays, except for:
A. *Deaths of Schwann cells in a distal segment
B. Disintegration of endings of nervous fibers
C. Tigrolysis
D. Destructions of myelin
E. Phagocytosis of fragments of the damaged nervous fibers

In conditional experiment action of toxic substance strengthens the mechanism of transfer of a


nervous pulse. What structure provides performance of this function?
A. *Neurofibril.
B. Neurolemma.
C. Synapse.
D. Mitochondrion.
E. Nissl bodies.

The traumatic damage of the nervous filaments is accompanied by the damage of axial cylinders,
disintegration of myelin. What nervous structures take part in reconstruction of myelin on
regeneration?
A. Perineurium.
B. Ependimocytes.
C. Neurolemmocytes (Schwann's cells).
D. Endoneurium.
E. Astrocytes.
Nerve system

235. One of sections of central nervous system has layer wise arrangement of neurocytes.
Among them there are cells of the following forms: stellate, fusiform, horizontal, pyramidal.
What section of central nervous system is this structure typical for?
A *Cortex of cerebrum
B Spinal cord
C Cerebellum
D Medulla oblongata
E Hypothalamus

237. Microscopic examination of a CNS body revealed the gray matter with three layers of
neurons, namely molecular, ganglionic and granular layer. What are the neurons constituting the
second layer?
A *Piriform
B Basket
C Small stellate
D Large stellate
E Granule cells

238. Microscopic analysis of a specimen revealed an organ of nervous system that consists of
pseudounipolar neurons covered with glial and connective tissue membranes. Determine this
organ:
A *Spinal ganglion
B Vegetative ganglion
C Spinal cord
D Cerebellum
E Cortex of cerebrum

239. A patient had a trauma that led to the injury of front spinal roots. Denote the damaged
structures:
A *Axons of motoneurons and lateral horn neurons
B Central processes of spinal ganglion neurons
C Peripheral processes of spinal ganglion neurons
D Axons of lateral horn neurons
E Axons of motoneurons

240. As a result of a trauma a patient has damaged frontal spinal roots. What structures are likely
to be affected?
A *Axons of the motoneurons and axons of the lateral horn neurons
B Central processes of the sensory neurons of the spinal ganglions
C Peripheral processes of the sensory neurons of the spinal ganglions
D Axons of the lateral horn neurons
E Dendrites of the spinal ganglion neurons

241. A ventral root of spinal cord was damaged as a result of a trauma. The following processes
of the following neurons were damaged:
A *Axons of motor neurons
B Dendrites of motor neurons
C Axons of sensory neurons
D Dendrites of sensory neurons
E Dendrites of internuncial neurons

243. As a result of a trauma a patient has damaged anterior roots of spinal cord. What structures
have been affected?
A *Axons of motoneurons and axons of neurons of lateral horns
B Central processes of sensitive neurons of spinal ganglions
C Peripheral processes of sensitive spinal ganglions
D Axons of neurons of lateral horns
E Dendrites of neurons of spinal ganglions

245. One of departments of CNS has a layer with location of neurocytes, among which one can
distinguish cells of different shapes: star-shape, fusiform, horizontal, pyramid. What department
of the nervous system corresponds to such structure?
A *Cortex of large hemispheres
B To the spinal cord
C To the cerebellum
D To the medulla oblongata
E To the hypothalamus

246. In a histological specimen of the department of the nervous system, a layer with location of
neurocytes, among which one can distinguish cells of different shapes: star-shape, fusiform,
horizontal, pyramid. What department of the nervous system is it?
A * Cortex of large hemispheres of cerebrum
B Cortex of cerebellum
C Spinal ganglion
D Vegetative ganglion
E Spinal cord

247. As a result of trauma of a man of 47 years, the anterior roots of spinal cord were damaged.
The processes of what neurons are damaged?
A * Axons of motor neurons of somatic and vegetative nuclei
B Axons of sensitive pseudounipolar neurons
C Dendrites of sensitive pseudounipolar neurons
D Dendrites of motor and axons of nuclei of lateral columns
E Dendrites and axons of sensitive pseudounipolar neurons

Alcoholic intoxication is accompanied by disturbed motor coordination and equilibrium due to


the damage caused to structural elements of the cerebellum. Functional disturbance of the
following cells occurs in the first place:
A * Pyriform cells
B Basket cells
C Granule cells
D Stellate cells
E Fusiform cells

Parenchyma of an organ is composed of pseudounipolar neurons localized under the capsule of


connective tissue. Central place belongs to nerve fibers. Name this organ:
A. *Spinal ganglion
B. Sympathetic ganglion
C. Intramural ganglion
D. Nerve trunk
E. Spinal cord

3. A report from a hospital pathology laboratory indicates that a microscope slide with a small
specimen of neural tissue contains “numerous GFAP-positive” cells. What is the most likely
source of this specimen?
a. A region of white matter
b. A sensory ganglion
c. An autonomic ganglion
d. *A region of gray matter
e. Pia mater

4. In the choroid plexus water from capillaries is transported directly into the cerebrospinal fluid
by what structure(s)?
a. *Ependyma
b. Astrocytes
c. Cells of the arachnoid mater
d. Lining of the central canal
e. Microglial cells

5. What term applies to collections of neuronal cell bodies (somata) in the central nervous
system?
a. Ganglia
b. Neuroglia
c. Nodes
d. White matter
e. *Nuclei

6. Which structure contains trabeculae around which cerebrospinal fluid (CSF) flows?
a. *Arachnoid mater
b. Ependyma
c. Dura mater
d. Pia mater
e. Gray matter

7. Which of the following is a characteristic of the connective tissue layer that surrounds
individual fascicles in large peripheral nerves?
a. A delicate region of connective tissue in contact with Schwann cells
b. Called the dura mater
c. *Important as part of the blood-nerve barrier in the nerve
d. Rich in myelin
e. The thickest sheath of connective tissue in the nerve

10. A 2-year-old boy presents with hearing impairment, poliosis (a white shock of hair),
complete heterochromia and sectoral heterochromia, hypertelorism, a low hairline with eyebrows
that touch in the middle, white pigmentation of the skin, and suspected neurologic deficits. He is
diagnosed with Waardenburg syndrome with a mutation in the PAX-3 gene that affects neural
crest differentiation. Which of the following structures would most likely also be affected in this
patient?
a. Purkinje cells
b. Pyramidal neurons
c. Ventral horns of the spinal cord
d. Astrocytes
e. *Neurons and satellite cells of the spinal ganglion

Acetylcholine is the only neurotransmitter in which of the following regions of the nervous
system?
A Central nervous system
B Presynaptic sensory cortex
C *Myoneural junctions
D Postganglionic sympathetic synapses
E Motor cortex

Sense organs

249. An infectious disease caused contractive activity of muscles that contract and dilate eye
pupil (paralytic state). What functional eye system was damaged?
A *Accommodative
B Dioptric
C Ancillary
D Photosensory
E Lacrimal apparatus
250. During an experiment the median part of an animal's cochlea was damaged. This resulted in
impaired perception of acoustic vibrations of the following frequency:
A *Medium
B Low
C High
D High and medium
E Low and medium

251. A patient has applied eye drops containing atropine which resulted in persistent mydriasis.
Which muscle was blocked?
A *Pupil-contracting
B Pupil-dilating
C Ciliate
D Rectus
E Oblique

252. A histological specimen presents a receptor zone of a sensoepithelial sense organ. Cells of
this zone are placed upon the basal membrane and include the following types: external and
internal receptor cells, external and internal phalangeal cell, stem cells, external limiting cells
and external supporting cell. The described receptor zone belongs to the following sense organ:
A *Acoustic organ
B Visual organ
C Gustatory organ
D Equilibrium organ
E Olfactory organ

253. A histological specimen of an eyeball shows a structure in form of a convexo-convex


formation connected with the ciliary body by the fibers of ciliary zonula and covered with a
transparent capsule. Specify this structure:
A *Crystalline lens
B Vitreous body
C Ciliary body
D Cornea
E Sclera

254. As a result of punctate retinal hemorrhage a patient lost ability to see objects in the center of
visual field. In what part of retina did the hemorrhage take place?
A *Yellow spot
B Ciliary part of retina
C Iris
D Blind spot
E Vascular membrane

257. The increased intraocular tension is observed in the patient with glaucoma. Secretion of
aqueous humor by the ciliar body is normal. Injury of what structure of the eyeball wall caused
the disorder of flow-out from the anterior chamber?
A *Venous sinus
B Ciliar body
C Choroid
D Ciliary muscle
E Back epithelium of cornea

258. Vitamin A deficit results in the impairment of twilight vision. Name the cells that have the
above-mentioned photoreceptor function:
A *Rod receptor cell
B Horizontal neurocytes
C Cone receptor cells
D Bipolar neurons
E Ganglion neurocytes

261. On an electronic photomicrograph, a cell of neural origin which has an epithelium of


mucous aeriferous ways was present. Distal part of peripheral process of the cell has a club-
shaped bulge with 10-12 cilia. What kind of cell is it?
A * Olfactory receptor cells
B Bipolar neuron of spinal ganglion
C Sensory epitheliocyte of organ of taste
D Rod visual cell
E Cone visual cell

262. After a boxer had trauma of the nose, violation of sense of smell is marked. Indicate which
cell's damage can result in anosmia?
A * Neurosensory cells
B Supporting epitheliocytes
C Basal epitheliocytes
D Ciliary epitheliocytes
E Microvillous epitheliocytes

263. On histological specimen the receptor zone of sensory-epithelial sense organ is determined.
The cells of this area lie on a basal membrane and include followings kinds: external and internal
sensory, external and internal phalange, pillar cells, external boundary and external supporting.
Indicate, what sense-organ this receptor zone belongs to:
A * To the organ of hearing
B To the organ of vision
C To the organ of taste
D To the organ of equilibrium
E To the olfactory organ

264. As a result of trauma of head of a man of 32 years, damage of crista ampullaris occurred.
Perception of what irritations will be violated?
A * Angular accelerations
B Vibrations
C Gravitation
D Linear accelerations
E Vibration and gravitation

265. The ciliary body of a patient was damaged. The function of what apparatus of eye will
suffer?
A * Accommodation
B Light-conducting
C Light-accepting
D Protective
E Trophic

266. On an electronic photomicrograph, a cell of neural origin is presented. Terminal part of


dendrite of this cell has a shape of cylinder and consists of 1000 closed membranous disks. What
kind of cell is it?
A * Rod visual cell
B Neuron of anterior horns of spinal cord
C Neuron of spinal ganglion
D Neuron of cortex of large hemispheres
E Cone visual cell
267. In a patient of 14 years, there is a violation of dusk vision. What vitamin is insufficiently
presented in this patient?
A* A
B B1
C B6
DC
E B12

268. As a result of trauma of nose of a man of 30 years, a mucosal layer that covers the overhead
part of superior shell was damaged. What consequences did it result in?
A * Imperceptions of odorous matters
B Violation of moistening of air
C Violation of secretory activity of goblet cells
D Violation of warming of air
E Violation of warming and moistening of air

269. At an inspection, oculist revealed that a patient does not distinguish a dark blue and green
color, at normal perception of other color gamut. With a disturbance of what structures of retina,
it connected with?
A * Cone visual cell
B Rod visual cell
C bipolar neurons
D Amacrine neurons
E Horizontal neurons

270. At the lack of vitamin of A has violation of dusk vision. Indicate cells which the noted
photoreceptor function belongs to.
A * Rod visual cell
B Horizontal neurons
C Cone visual cell
D Bipolar neurons
E Ganglion nervous cells

A 60-year-old patient has reduced perception of high-frequency sounds. What structures’


disorder of auditory analyzer caused these changes?
A *Main membrane of cochlea near the oval window
B Main membrane of cochlea near helicotrema
C Eustachian tube
D Muscles of middle ear
E Tympanic membrane

While shifting the gaze to the closely situated object the refracting power of eye’s optical
mediums will increase by 10 diopters. It results from changing of such eye structure:
A *Lens
B Cornea
C Vitreous body
D Liquid of the anterior chamber of eye
E Muscle that dilates pupil

A 25-year-old patient complained of the decreased vision. Accommodation disorders, dilated


pupil, lack of reaction for the light were revealed on examination. What muscles function is
disturbed?
A *Pupil narrowing muscle, ciliary
B Pupil dilating muscle, ciliary
C Inferior oblique muscle, ciliary
D Lateral rectus muscle, pupil narrowing
E Pupil narrowing and dilating muscles
After a 2 y.o. child has had flu, there appeared complaints about ear ache. A doctor revealed
hearing impairment and inflammation of the middle ear. How did the infection penetrate into the
middle ear?
A *Through the auditory tube
B Through foramen jugularis
C Through canalis caroticus
D Through atrium mastoideum
E Through canalis nasolacrimalis

A 60-year-old patient has impaired perception of high-frequency sounds. These changes were
caused by damage of the following auditory analyzer structures:
A *Main cochlea membrane near the oval window
B Main cochlea membrane near the helicotrema
C Eustachian tube
D Middle ear muscles
E Tympanic membrane

A male working as a blacksmith has been tested for auditory acuity. The tests revealed 50%
hearing loss in the low frequency range and a near-normal auditory acuity in the high-frequency
range. This condition has been caused by the damage to the following structures of the auditory
system:
A *Corti’s organ - closer to helicotrema
B Corti’s organ - closer to the oval foramen
C Median part of the Corti’s organ
D Muscles of the middle ear
E Eardrum

A patient complains of photoreception disorder and frequent acute viral diseases. He has been
prescribed a vitamin that affects photoreception processes by producing rhodopsin, the
photosensitive pigment. What vitamin is it?
A *Retinol acetate
B Tocopherol acetate
C Pyridoxine hydrochloride
D Cyanocobalamin
E Thiamine

An oculist has detected increased time of darkness adaptation of a patient’s eye. What vitamin
deficiency can cause such symptom?
A *А



ED

A person with vitamin A deficiency develops twilight vision disturbance. Name the cells that
fulfill this photoreceptor function:
A. *Rod cells
B. Horizontal cells of retina
C. Bipolar neurons
D. Cone cells
E. Ganglionic nerve cells

A hospital has received a 24-year-old man, who had received a penetrating wound to the eye,
which has caused the vitreous body to run out. As the result of this, retinal detachment occurred.
What retinal layer was tightly adherent to the vascular tunic of the eye and did not detach?
A. *Retinal pigment epithelium
B. Layer of rod sand cones
C. Ganglion cell layer
D. Outer nuclear layer
E. Inner nuclear layer

Cardiovascular system

271. Interlobular capillaries of a liver specimen have wide irregular lumen. Basal membrane is
absent in the major part of the capillary. What type of capillaries is it?
A *Sinusoid
B Visceral
C Somatic
D Precapillaries
E Postcapillaries

272. A histological specimen shows a blood vessel. Its inner coat is composed by endothelium,
subendothelium and internal elastic membrane. The middle coat is enriched with smooth
myocytes. Such morphological characteristics are typical for the following vessel:
A *Muscular-type artery
B Elastic-type artery
C Capillary
D Non-muscular vein
E Muscular-type vein

273. Obliterating atherosclerosis causes changes in the vessels of the lower extremities. A
histological specimen of such a vessel evidently presents both internal and external elastic
membranes, middle membrane contains a lot of myocytes. What vessel is affected in case of this
disease?
A *Artery of muscular type
B Artery of elastic type
C Artery of mixed type
D Vein with strongly developed muscles
E Lymph node

274. A specimen of the pia mater shows a vessel with no middle membrane in its wall, its outer
membrane adheres to the surrounding tissues, the inner membrane is made up of the basal
membrane and endothelium. Specify this vessel:
A * Unmuscular vein
B Muscular vein with weakly developed muscular elements
C Muscular artery
D Arteriola
E Mixed artery

275. Morphological examination revealed in histological specimen of biopsy material an


irregular-shaped vessel. Its middle membrane is formed by bundles of smooth myocytes and
layers of connective tissue. What type of vessel is it?
A *Vein of muscular type
B Artery of muscular type
C Lymphatic vessel
D Venule
E Arteriole

277. A specimen of pia mater includes a vessel whose wall doesn't have the tunica media, the
tunica externa is adherent to the surrounding tissues, the intima is composed of a basement
membrane and endothelium. What vessel is it?
A *Nonmuscular vein
B Muscular vein with underdeveloped muscular elements
C Muscular artery
D Arteriole
E Artery of mixed type
278. In the microspecimen of red bone marrow there were revealed multiple capillaries through
the walls of which mature blood cells penetrated. What type of capillaries is it?
A *Sinusoidal
B Fenestrational
C Somatical
D Visceral
E Lymphatic

280. A histological specimen of spleen shows a vessel with a wall consisting of endothelium and
subendothelial layer, median membrane is absent, exterior membrane inosculates with the layers
of spleen connective tissue. What vessel is it?
A *Vein of non-muscular type
B Vein of muscular type
C Artery of muscular type
D Arteriole
E Capillary

281. A histological specimen presents an artery. One of the membranes of its wall has flat cells
lying on the basal membrane. What type of cells is it?
A *Endothelium
B Mesothelium
C Smooth myocytes
D Fibroblasts
E Macrophages

282. In specimen of a tubular organ, stained with orcein, it is discovered about 50 thick
membranes which have a wave form and makes the base of the middle layer of this organ. Which
organ is this?
A *Aorta
B Artery of muscular type
C Esophagus
D Trachea
E Wall of heart

283. In the maternity department, doctors were not able to notice the first breathing in a
newborn. By performing medical analysis, it was stated that the reason of death is that aeriferous
ways are free, but lungs did not recoil. What is the most reliable reason of unrevealed lungs in
this case?
A * Absence of surfactant
B Spasm of bronchi
C Rupture of bronchi
D Bulge of pleura
E Enlargement of alveolus.

284. On microscopic specimen is presented an organ of the circulatory system. One of its
envelopes is built from fibers which anastomose one with another. These fibers are made up of
cells which are united one with another with the intercalated disks. What it is an organ?
A *Heart
B Vein of muscular type
C Artery of muscular type
D Artery of elastic type
E Arteriole

285. On a histological specimen, stained with orcein, on the middle envelope of vessel it was
discovered 40 to 60 elastic membranes. Name this vessel.
A *Artery of elastic type
B Artery of muscular type
C Artery of the mixed type
D Vein of muscular type
E Vein of non-muscular type

286. On a histological specimen, it is observed a vessel, the wall of which consists of


endothelium, basal membrane and loose connective tissue. Name the type of vessel?
A * Vein of non-muscular type
B Artery
C Vein of muscular type
D Hemocapillary
E Lymphocapillary

287. On histological specimen a blood vessel is observed. An internal envelope consists of


endothelia, subendothelia and internal elastic membrane. The middle envelope enriched by
smooth muscle cells. Indicate the vessel
A * Arteries of muscular type
B Arteries of elastic type
C Capillary
D Veins of non-muscular type
E Veins of muscular type

288. Vessels, which begin blindly, have been observed in histological specimen. They have the
appearance of flattened endothelial tubes, which does not contain a basal membrane and
pericytes, the endothelia of these vessels is fixed by strop filaments to the collagen fibers of
connective tissue. What is it vessels?
A *Lymphoid capillaries
B Hemocapillary
C Arterioles
D Venules
E Arterio-venous anastomoses

289. On histological specimen a blood vessel is presented. An internal envelope consists of


endothelia, subendothelia and internal elastic membrane. Smooth muscle cells predominate in a
middle envelope. An external envelope consists of loose connective tissue. Indicate the vessel.
A *Artery of muscular type
B Arteries of elastic type
C Arteries of mixed type
D Veins of muscular type
E Veins of non-muscular type

290. On specimen a blood vessel is presented. An internal envelope is presented by endothelium


and subendothelium, middle envelope - by the bundles of smooth muscle cells, layers loose
connective tissue. An external envelope is strongly developed and formed by envelope and
separate smooth muscle cells. What vessel does have this morphological description?
A *Veins of muscular type
B Artery of muscular type
C Vein of non-muscular of type
D Artery of the mixed type
E Artery of elastic type

291. On specimen in one of vessels of the microcirculatory bed, the middle envelope is formed
by 1-2 layers of smooth muscle cells, which are located alone and have a spiral direction. An
external envelope is represented by a thin layer of loose connective tissue. Indicate the type of
vessel.
A *Arteriole
B Venule
C Capillary
D Postcapillary venule
E Arterio-venous anastomoses

293. There are a lot of morphological differences in the middle envelope of the walls of blood
vessels. What is the reason for these differences?
A.*Hemodynamic conditions.
B. Influence of organs of the endocrine system.
C. Regulation by central nervous system.
D. Inductive influence of neurons of vegetative ganglion.
E. High concentration of catecholamines in blood.

295. An artery of large caliber expands during a systolic stretch and returns to the original state
during diastole, provides stability of blood stream. By the presence of what elements of wall of
vessel can it be explained?
A *Elastic fibers
B Muscular fibers
C Reticular fibers
D Collagen fibers
E By plenty of fibroblasts

296. A patient suffered myocardial infarction for some time, after which the morphological
integrity of the heart wall was restored. What tissue is necessary for the regeneration?
A*Connective
B Smooth muscle
C Striated muscular
D Epithelial
E Nervous

Examination of an isolated cardiomyocyte revealed that it didn’t generate excitation impulses


automatically. This cardiomyocyte was obtained from:
A *Ventricles
B Sinoatrial node
C Atrioventricular node
D His’ bundle
E Purkinje’s fibers

Short-term physical activity resulted in reflex amplification of heart rate and raise of systemic
arterial pressure. What receptors activation was the main cause of pressor reflex realization?
A *Proprioreceptors of active muscles
B Vascular chemoreceptors
C Vascular volume receptors
D Vascular baroceptors
E Hypothalamus thermoreceptors

Atria of an experimental animal were superdistended with blood, which resulted in decreased
reabsorption of Na+ and water in renal tubules. This can be explained by the influence of the
following factor on kidneys:
A *Natriuretic hormone
B Aldosterone
C Renin
D Angiotensin
E Vasopressin

One of the coats of a hollow organ has anastomotic fibers with nuclei. The fibers consist of cells
that form intercalated disks at the places of contact. What tissue forms this coat?
A *Cross-striated cardiac muscle
B Cross-striated skeletal muscle
C Unstriped muscle
D Loose fibrous connective tissue
E Dense irregular connective tissue

Investigation of an isolated cardiac myocyte determined that it does not generate excitation
impulses automatically, which means this cardiac myocyte was obtained from the following
cardiac structure:
A. *Ventricles
B. Sinoatrial node
C. Atrioventricular node
D. His’ bundle
E. Purkinje’s fibers

Histologic preparation stained with orcein demonstrates from 40 to 60 fenestrated elastic


membranes within the middle coat of the vessel. Name this vessel:
A. *Elastic artery
B. Muscular artery
C. Mixed type artery
D. Muscular vein
E. Nonmuscular vein

An electron micrograph shows a small vessel with endothelial layer but without basement
membrane and pericytes; anchoring fibrils are present. Name this vessel:
A. *Lymph capillary
B. Arteriole
C. Venule
D. Sinusoid hemocapillary
E. Visceral hemocapillary

Central endocrine organs

199. The aim of the morphological study was to investigate an endocrine gland with parenchyma
consisting of epithelium and neural tissue. In the epithelial trabeculae the study revealed two
types of cells: chromophile and chromophobe. Identify this organ:
A *Pituitary gland
B Adrenal gland
C Hypothalamus
D Thyroid gland
E Parathyroid gland

304. A 37-year-old patient complains about permanent thirst, poor appetite. He drinks 9 l water
per day. Daily diuresis is increased, urine is colorless, its relative density is 1,005. The most
probable cause of this pathology development is damage of:
A *Hypothalamic nuclei
B Epithelium of renal tubuli
C Adenohypophysis
D Epiphysis
E Basal membrane of glomerular capillaries

310. X-ray examination of skull base bones revealed enlargement of sellar cavity, thinning of
anterior clinoid processes, destruction of different parts, destruction of different parts of sella
turcica. Such bone destruction might be caused by a tumor of the following endocrine gland:
A *Hypophysis
B Epiphysis
C Thymus gland
D Adrenal glands
E Thyroid gland

311. An endocrinal gland with parenchyma consisting of epithelium and neural tissue is under
morphological examination. Epithelial trabecules have two types of cells: chromophilic and
chromophobic. Identify this organ:
A *Hypophysis
B Adrenal glands
C Hypothalamus
D Thyroid gland
E Parathyroid gland

312. For morphological research an endocrine gland is presented. The parenchyma of this gland
consists of epithelium and nervous tissue. In epithelial trabecules it appears 2 types of cells:
chromophilic and chromophobic. Define this organ.
A *Hypophysis
B Adrenal gland
C Hypothalamus
D Thyroid
E Parathyroid gland

313. During births a woman had weak contraction of myocytes of uterus. To insufficiency of
what hormone of hypothalamus is it possible to explain this state?
A * Oxytocin
B Foliliberin
C Prolaktoliberin
D Somatoliberin
E Vasopressin

315. A woman during births had a weak contractile activity of uterus. What hormone of
hypothalamus can increase contractile activity of uterus in this situation?
A * Oxytocin
B Vasopressin
C Liberin
D Statin
E Antidiuretic hormone

316. At an experiment the axons of neurosecretory cells of supraoptic nuclei of hypothalamus


were cut. The accumulation of what hormone in a hypophysis will be broken?
A * Vasopressin
B Somatotropic
C Prolactotropic
D Adrenocorticotropic
E Lipotropic

324. The parenchyma of adenohypophysis is presented by trabecules, formed by glandular cells.


Among adenocytes there are cells with granules, which are stained with basic dyes and contain
glycoproteins. What cells is it?
A *Gonadotropocytes, thyrotropocytes
B Somatotropocytes
C Melanotropocytes
D Mamotropocytes
E Chromophobic

326. At X-ray examination of bones of the base of skull it was revealed enlargement of cavity of
Sella turcica ("Turkish saddle"), thinning of anterior inclined projection, destruction of different
parts of Sella turcica. Tumor of what endocrine gland could lead to such destruction of bones?
A * Hypophysis
B Epiphysys
C Thymus
D Adrenal gland
E Thyroid gland
327. A patient has an elevated excretion of urine during the day. Due to the lack of secretion of
what hormone of hypothalamus is it possible to explain this phenomenon?
A * Vasopressin
B Oxytocin
C Liberin
D Statin
E Thyriod

328. From the ectodermal epithelium of lining of overhead part of mouth fossula of embryo of
man the pocket of Rathke is formed, which is directed to the basis of future cerebrum. What does
develop from this embryonic bud?
A *Adenohypophysis
B Neurohypophysis
C Eminentia medialis
D Hypophyseal stalk
E Anterior hypothalamus

A man after 1,5 liters of blood loss has suddenly reduced diuresis. The increased secretion of
what hormone caused such diuresis alteration?
A *Vasopressin
B Corticotropin
C Natriuretic
D Cortisol
E Parathormone

A person felt thirsty after staying under the conditions of hot weather for a long time. Signals of
what receptors caused it first of all?
A *Osmoreceptors of hypothalamus
B Sodium receptors of hypothalamus
C Osmoreceptors of liver
D Glucoreceptors of hypothalamus
E Baroreceptors of aortic arch

Examination of a patient revealed overgrowth of facial bones and soft tissues, tongue
enlargement, wide interdental spaces in the enlarged dental arch. What changes of the hormonal
secretion are the most likely?
A * Hypersecretion of the somatotropic hormone
B Hyposecretion of the somatotropic hormone
C Hypersecretion of insulin
D Hyposecretion of thyroxin
E Hyposecretion of insulin

A 32-year-old patient consulted a doctor about the absence of lactation after parturition. Such
disorder might be explained by the deficit of the following hormone:
A *Prolactin
B Somatotropin
C Vasopressin
D Thyrocalcitonin
E Glucagon

Examination reveals no disorders of carbohydrate metabolism. These abnormalities might be


caused by the dysfunction of the following endocrine gland:
A *Neurohypophysis
B Adenohypophysis
C Islets of Langerhans (pancreatic islets)
D Adrenal cortex
E Adrenal medulla
A 20-year-old patient complains of morbid thirst and polyuria (diuresis up to 10 daily). Glucose
concentration in blood is normal but it is absent in urine. The patient has been diagnosed with
diabetes insipidus. What hormonal drug is the most appropriate for management of this disorder?
A *Vasopressin
B Cortisol
C Thyroxin
D Oxytocin
E Insulin

In the course of an experiment adenohypophysis of an animal has been removed. The resulting
atrophy of thyroid gland and adrenal cortex has been caused by deficiency of the following
hormone:
A *Tropic hormone
B Thyroid hormones
C Somatotropin
D Cortisol
E Thyroxin

Corticosteroid hormones regulate the adaptation processes of the body as a whole to


environmental changes and ensure the maintenance of internal homeostasis. What hormone
activates the hypothalamopituitary-adrenal axis?
A *Corticoliberin
B Somatoliberin
C Somatostatin
D Corticostatin
E Thyroliberin

Secretion in what neuroendocrine cell is controlled directly by neural activity and involves a
hormone that generally slows metabolic activity at night?
a. Pituicyte
b. Melanocyte
c. Herring body of the neurohypophysis
d. Chromaffin cell
e. *Pinealocyte

Peripheral endocrine organs

297. A patient with Itsenko-Cushing syndrome has persistent hyperglycemia and glycosuria,
hypertension, osteoporosis, obesity. Increased synthesis and hypersecretion of the following
hormone will be observed in this case:
A *Cortisol
B Adrenaline
C Glucagon
D Thyroxin
E Aldosterone

298. A 5-month-old boy was hospitalized for tonic convulsions. He has a life-time history of this
disease. Examination revealed coarse hair, thinned and fragile nails, pale and dry skin. In blood:
calcium - 1,5 millimole/l, phosphor - 1,9 millimole/l. These changes are associated with:
A *Hypoparathyroidism
B Hyperparathyroidism
C Hyperaldosteronism
D Hypoaldosteronism
E Hypothyroidism

300. Microscopic examination of a parenchymatous organ revealed that its epithelial cords
formed glomerular, fascicular and reticular zones. The central part of the organ was presented by
accumulations of chromaffin cells. Specify this organ:
A *Adrenal gland
B Thyroid gland
C Epiphysis
D Liver
E Hypophysis

301. A 40-year-old female patient has enlarged thyroid gland. On palpation the gland is dense, its
surface is slightly tuberous. Histological examination of gland sample revealed diffuse
infiltration of tissue by the cells, formation of lymphoid follicles. What disease is it?
A *Autoimmune thyroiditis
B Endemic goiter
C Sporadic goiter
D Diffuse toxic goiter
E Riedel's disease

302. After a surgical procedure an experimental animal died from intense convulsions. What
endocrinal glands were extracted?
A *Parathyroid
B Thyroid
C Adrenal
D Ovaries
E Testicles

303. Clinical examination of a female patient revealed reduction of basal metabolism by 40\%,
gain in body mass, drop of body temperature, face puffiness, sexual dysfunctions, inertness and
apathy, lowered intelligence. These symptoms are caused by dysfunction of the following
endocrine gland:
A *Hypofunction of thyroid gland
B Hypofunction of parathyroid glands
C Hypophysis hyperfunction
D Epiphysis hypofunction
E Hyperfunction of thyroid gland

305. A patient ill with adenoma of glomerular zone of adrenal cortex (Conn's disease) has arterial
hypertension, convulsions, polyuria. What is the main factor in the pathogenesis of these
disturbances?
A *Aldosterone hypersecretion
B Aldosterone hyposecretion
C Catecholamines hypersecretion
D Glycocorticoids hypersecretion
E Glycocorticoids hyposecretion

306. A patient has been given high doses of hydrocortisone for a long time. This caused atrophy
of one of the adrenal cortex zones. Which zone is it?
A *Fascial
B Glomerular
C Reticular
D Glomerular and reticular
E -

307. Parodontitis is treated with calcium preparations and a hormone that stimulates tooth
mineralization and inhibits tissue resorption. What hormone is it?
A *Calcitonin
B Parathormone
C Adrenalin
D Aldosterone
E Thyroxine
308. A 9 y.o. boy was admitted to the endocrinological department. This boy has already had
several fractures of hist extremities due to bone brittleness. The function of the following
endocrinal glands (gland) is disturbed:
A *Parathyroid
B Thyroid
C Thymus
D Adrenal
E Epiphysis

309. Kidneys of a man under examination show increased resorption of calcium ions and
decreased resorption of phosphate ions. What hormone causes this phenomenon?
A *Parathormone
B Thyrocalcitonin
C Hormonal form D3
D Aldosterone
E Vasopressin

314. On a histological specimen of the parenchymal organ. It has a structural and functional unit
of which is a follicle. The wall of follicle is formed by the cells of cuboidal shape; the cavity of
follicle is filled with colloid. What organ is presented on specimen?
A * Thyroid gland
B Salivary gland
C Hypophysis
D Ovary
E Testis

317. The high doses of hydrocortisone were used for a long time by the patient, as a result of
this, atrophy of one of areas of adrenal glands cortex occurred. What area is it?
A * Fasiculata
B Glomerulosa
C Reticularis
D Glomerulosa and reticularis
E-

318. At a patient of 30 years the hyperfunction of thyroid gland was revealed. What shape does
thyrocytes of follicles have?
A * Columnar
B Poligonal
C Flat
D Fusiform
E Cuboidal

319. At a patient of 42 after the operation of resection of thyroid gland, cramps appeared. A
facilitation came at infusion of calcium. Dysfunction of what glands does cause this state?
A *Parathyroid glands
B Adrenal glands
C Ovaries
D Hypophysis
E Epiphysis

320. On the specimen of one of endocrine glands the rounded structures of different sizes are
seen, the wall of which is formed by one layer of epithelial cells on basal membrane. Inwardly
these structures contain homogeneous noncellular mass. What gland is it?
A * Thyroid gland
B Adrenal gland
C Parathyroid gland
D Anterior hypophysis
E Posterior hypophysis
321. It is known that aldosterone regulates maintenance of sodium in an organism. What cells of
adrenal gland does produce this hormone?
A *Cells of glomerular zona
B Epinephrocytes
C Cells of the reticular zone
D Cells of fascicular zone
E Norepinephrocytes

322. Patient of 40 years was investigated by endocrinologist. Patient suffers from insufficiency
of adrenal gland, which realizes in decrease of aldosterone in blood. The function of what cells
of adrenal gland cortex is broken?
A * Cells of glomerular zona
B Cells of fascicular zone
C Cells of the reticular zone
D Cells of sudanophobic area
E Cells of X-zone

323. Parenchymal organ is presented in histological specimen; superficial layer of cortex is


presented by glomeruli, which are composed of endocrinocytes. To what organ does this
morphological specimen belong?
A * Adrenal gland
B Lymphatic node
C Spleen
D Thyroid
E Ovary

325. In the wall of follicles and in the interfollicle layers of connective tissue on territory of
thyroid gland large endocrinocytes are situated, secretory granules of which are osmio- and
argyrophilic. Name these cells.
A *Calcitoninocytes
B Thyrocytes
C Parathyrocytes
D Pinealocytes
E Pituicytes

Some diseases reveal symptoms of aldosteronism with hypertension and edema due to sodium
retention in the organism. What organ of the internal secretion is affected on aldosteronism?
A *Adrenal gland
B Testicle
C Ovaries
D Pancreas
E Hypophysis

A patient suffering from thyrotoxicosis symptoms of vegetoasthenic syndrome was revealed.


What of the following would show the histological appearance of a thyroid gland being
stimulated by thyroid-stimulating hormone (TSH)?
A * Columnar-shaped follicular cells
B Decreased numbers of follicular cells
C Increased numbers of parafollicular cells
D An abundance of colloid in the lumen of the follicle
E Decreased numbers of parafollicular capillaries

A 19-year-old female suffers from tachycardia in rest condition, weight loss, excessive sweating,
exophthalmos and irritability. What hormone would you expect to find elevated in her serum?
A *Thyroxine
B Cortisol
C Mineralocorticoids
D ACTH
E Insulin

An aged man had raise of arterial pressure under a stress. It was caused by activation of:
A *Sympathoadrenal system
B Parasympathetic nucleus of vagus
C Functions of thyroid gland
D Functions of adrenal cortex
E Hypophysis function

A concentrated solution of sodium chloride was intravenously injected to an animal. This caused
decreased reabsorption of sodium ions in the renal tubules. It is the result of the following
changes of hormonal secretion:
A *Aldosterone reduction
B Aldosterone increase
C Vasopressin reduction
D Vasopressin increase
E Reduction of atrial natriuretic factor

A middle-aged man went to a foreign country because he had been offered a job there. However,
he had been unemployed for quite a long time. What endocrine glands were exhausted most of
all in this man?
A * Adrenal glands
B Parathyroid glands
C Seminal glands
D Substernal gland
E Thyroid gland

A girl has been diagnosed with adrenogenital syndrome (pseudohermaphroditism). This


pathology is caused by hypersecretion of the following adrenal hormone:
A *Androgens
B Estrogens
C Mineralocorticoids
D Glucocorticoids
E Catecholamines

People adapted to high external temperatures have such peculiarity: profuse sweating isn’t
accompanied by loss of large volumes of sodium chloride. This is caused by the effect of the
following hormone upon the perspiratory glands:
A *Aldosterone
B Vasopressin
C Cortisol
D Thyroxin
E Natriuretic

Inhabitants of territories with cold climate have high content of an adaptive thermoregulatory
hormone. What hormone is meant?
A *Thyroxin
B Insulin
C Glucagon
D Somatotropin
E Cortisol

A 19-year-old male was found to have an elevated level of potassium in the secondary urine.
These changes might have been caused by the increase in the following hormone level:
A *Aldosterone
B Oxytocin
C Adrenaline
D Glucagon
E Testosterone

Presented is the biopsy material of an organ consisting of saccule-shaped rounded structures of


varying size. Inside these structures there is a gel-like non-cellular substance - colloid; structure
walls are composed of one layer of cuboidal cells that lay on the basement membrane. Between
the saccules there is connective tissue with vessels. Name this organ:
A * Thyroid gland
B Pancreas
C Parotid gland
D Thymus
E Parathyroid gland

A patient with signs of osteoporosis and urolithiasis has been admitted to the endocrinology
department. Blood test has revealed hypercalcemia and hypophosphatemia. These changes are
associated with abnormal synthesis of the following hormone:
A * Parathyroid hormone
B Calcitonin
C Cortisol
D Aldosterone
E Calcitriol

Histological specimen demonstrates a parenchymal organ with cortical and medullary


substances. The cortical substance is composed of bands of epithelial cells with capillary blood
vessels between them. The bands form three zones. The medullary substance consists of
сhromaffin cells and venous sinusoids. What organ can be characterized by these morphological
features?
A. *Adrenal gland
B. Kidney
C. Lymph node
D. Thymus
E. Thyroid gland
Skin

329. Histological study of a microslide of human skin found only dense irregular connective
tissue. Which layer of this organ was analyzed?
A *Reticular dermis
B Papillary dermis
C Subcutaneous adipose tissue
D Epidermis
E Basal layer of epidermis

330. A patient complains of dryness of head skin, itching, fragility and loss of hair. After
examination he was diagnosed with seborrhea. Disturbed activity of which cells caused this
condition?
A *Cells of sebaceous glands
B Cells of sudoriferous glands
C Epithelial cells
D Adipocytes
E Melanocytes

331. A scheme presents an exocrinous gland that has unbranched excretory duct with a terminal
part in form of a saccule opening into the duct. How is this gland called according to the
morphological classification of exocrinous glands?
A *Simple unbranched alveolar
B Compound branched alveolar
C Simple branched tubular
D Compound unbranched alveolar
E Compound unbranched alveolar tubular

332. Study of fingerprints (dactylography) is used by criminalists for personal identification as


well as for diagnostics of genetic abnormalities, particularly Dawn's disease. What layer of skin
determines individuality of fingerprints?
A *Dermopapillary
B Horny
C Reticular
D Clear (stratum lucidum epidermidis)
E Basal

333. The study of imprints of epidermis of fingers of hands (dactylography) is used by criminal
lawyers for identification of a person, and also for diagnostics of genetic anomalies, in particular
Down syndrome. What layer of skin determines the individuality of imprints?
A * Papillary
B Horny
C Reticular
D Translucent
E Basal

334. At an embryo the processes of segmentation of dorsal mesoderm and formation of somites
were broken. In what part of skin violations of development are possible?
A *Derma
B Hair
C Sebaceous
D Epidermis
E Sweat glands

335. In a biopsy material of the skin, in the epidermis were revealed cells with sprouts with
granules of brown color in cytoplasm. What kind of cell is it?
A * Melanocytes
B Intraepidermal macrophages
C Keratinocytes
D Merkel cells
E Lymphocytes

336. The cells of basal layer of epidermis suffered under influence of radiation. What function of
epidermis will attenuate or upset foremost?
A *Regenerative
B Protective
C Barrier
D Absorption
E Dielectric

337. The terminal portions of apocrine sweat-glands are contained by myoepithelial cells. What
is the function of these cells?
A *Contractile
B Secretory
C Protective
D Regeneration
E Supporting

338. A patient complains of a dryness of skin of head, itch, fragility and fall of hair. At an
inspection the set diagnosis: Seborrheic dermatitis. With violation of activity of what cells is it
connected?
A * Cells of sebaceous gland
B Cells of sweat glands
C Epitheliocytes
D Adipocytes
E Melanocytes

339. At the patient of 30 years the malignant tumor of skin was found out. What cells of
epidermis do take part in an immune response?
A * T-lymphocytes
B Keratinocytes
C Keratinocytes and Merkel cells
D Merkel cells
E Cells of spinous layer

340. The trauma of skin happened with the damage of layer of derma. Due to activity of what
cells will the regeneration of this layer happen?
A *Fibroblasts
B Macrophages
C Lymphoblasts
D Tissue basophils (mast cells)
E Plasmocytes

341. The method of dactylography, which is widely used in medico-legal examination based on
that the nipple layer of derma determines strictly an individual picture of the surface of skin.
What tissue does form this layer of derma?
A * Loose connective tissue
B Dense regular connective tissue
C Dense irregular connective tissue
D Reticular tissue
E Adipose tissue

342. On a histological specimen an organ of the stratified type structure, covered by a multi-
layered flattened keratinized epithelium is presented. Under the basal membrane of the
epithelium there is loose connective tissue which forms papillae. Located below is the dense
irregular connective tissue is which forms the reticular layer. What organ has this morphological
signs?
A * Skin
B Neck of uterus
C Tonsils
D Tong
E Esophagus

343. At the man of 53 it was revealed a malignant tumor of the pericardium. Which type of
epithelium is the source of this tumor?
A *Simple squamous
B Simple pseudostratified ciliary
C Transitional
D Stratified keratinized
E Stratified non-keratinized

344. On a histological specimen is a biopsy material of the epidermis of the skin. The skin is that
of a healthy adult man. In a basal layer it was evidenced cells which were dividing. What process
do these cells provide?
A *Physiological regeneration
B Differentiation
C Adaptation
D Reparative regeneration
E Apoptosis

345. On a limited area of epidermis as a result of trauma all layers up to a growth are absent.
What is the name of cells which will provide the regeneration?
A *Basal cells
B Spinous cells
C Granular cells
D Spinous and granular cells of the neighboring region
E Cells of transparent layer of the neighboring region

346. During an experiment, the tight junctions between epitheliocytes were destroyed. What
function of epithelium will suffer?
A *Mechanical
B Absorptive
C Vitamin D-producing
D Secretory
E Excretory

347. With age, skin wrinkles and folds appear. Changes in what structures of the skin mainly
cause this state?
A * Elastic fibers
B Collagen fibers
C Epidermis
D Ground substance
E Hypoderm

348. At an experiment on the embryo of a frog, the external embryonic layer- ectoderm was
destroyed. What morphological structure of this embryo will not develop in future?
A *Epidermis.
B Somites
C Nephrotome
D Splanchnotome
E Bone tissue

Human skin has a high breaking strength. It is known that the skin consists of epithelial tissue
and two kinds of connective tissue. Which of the following tissues provides the skin strength?
A *Irregular dense connective tissue
B Stratified squamous epithelium
C Loose connective tissue
D Single-layer epithelium
E Transitional epithelium

In an embryo the process of dorsal mesoderm segmentation and somite formation is disturbed.
What part of skin will probably have developmental abnormalities?
A *Dermis
B Hair
C Sebaceous glands
D Epidermis
E Perspiratory glands

A 12-year-old patient has white no pigmented spots on the skin. The spots appeared after the
patient became 10 years old, and they constantly grow.
A *Melanocytes
B Adipocytes
C Fibrocytes
D Plasmocytes
E Labrocytes

Digestive system 1

299. Examination of a microspecimen made of an unknown organ revealed some acini that
contained 10-15 cone cells with basophilic cytoplasm, round nucleus and well developed
granular endoplasmic reticulum. An acinus is surrounded by a basal membrane with
myoepithelial cells localized in its splitting. What organ is the slice made of?
A *Parotid gland
B Pancreas
C Lungs
D Sublingual gland
E Liver

349. A microspecimen of parotid gland presents secretory acines with serous cells that synthesize
mostly enzymes. According to the chemical composition classification, the parotid gland relates
to the following glands:
A *Serous
B Mucous
C Seromucous
D Enzymatic
E -

353. A child damaged the lateral surface of his tongue. What lingual papillas are most likely to
be damaged?
A *Foliate
B Conic
C Vallate
D Filiform
E Fungiform

358. A histological specimen represents a structure of the oral cavity, which is formed by bone
tissue. It is covered by mucous membrane consisting of keratinizing stratified squamous
epithelium. The structure has fatty, glandular and marginal zone. In all parts of the lamina
propria the collagen fibers form thick bundles that penetrate deep into the periosteum. What kind
of structure is it?
A *Hard palate
B Gingiva
C Lip
D Cheek
E Tongue

393. In histological specimen of organ of oral cavity it was evident, that anterior surface is
covered by stratified squamous non-keratinized epithelium, and posterior surface - by a ciliary
epithelium. What organ is it?
A * Soft palate
B Gums
C Hard palate
D Lip
E Cheek

394. At a patient who had acute pancreatitis, analyses were done and it was seen that the
excretory acinocytes were damaged. What cells will provide their renewal?
A* Cells of the intercalated ducts
B Cells of Langerhans islets
C Cells of interlobular ducts
D Cells of stroma of gland
E Endothelium of vessels

Histologic specimen of a tooth slice shows a tissue consisting of intercellular substance


permeated with tubules, in which cellular processes of odontoblasts are situated. What tissue is
presented in
A *Dentin
B Enamel
C Pulp
D Cement
E Periodontium

378. On a histological specimen, a transverse section through the wall of hollow organ, the
mucus envelope of which is covered by stratified squamous non-keratinized epithelium is seen.
What organ is it?
A*Esophagus
B Duodenum
C Colon
D Uterus
E Appendix

389. On the histological specimen of submandibular salivary gland around terminal portions and
excretory ducts myoepitheliocytes are observed. What tissue do these cells belong to?
A * Muscular
B Epithelial
C Nervous
D Connective with the special properties
E Loose connective

There is the change of teeth at the 6-8-year-old children: deciduous are replaced by permanent.
What embryonic tissues are the sources of formation of permanent teeth tissues?
A *Ectodermal epithelium of a tooth plate and mesenchyme
B Endodermal epithelium of a tooth plate and mesenchyme
C Mesodermal epithelium and mesenchyme
D I, II brachial arches
E Endodermal epithelium and mesoderm

The reason of occurrence of some diseases of an oral cavity is connected with structural
peculiarities of its mucous membrane. What morphological attributes characterize these features?
A *No muscularis mucosa, stratified squamous epithelium
B Transitional epithelium, no submucosa
C Simple columnar ciliated epithelium
D Well developed muscularis, no submucosa
E Transitional epithelium, no muscularis mucosa

Protective function of saliva is based on several mechanisms, including the presence of enzyme
that has bactericidal action and causes lysis of complex capsular polysaccharides of
staphylococci and streptococci. Name this enzyme:
A *Lysozyme
B Alpha-amylase
C Oligo-1,6-glucosidase
D Collagenase
E Beta-glucuronidase

The patient’s saliva has been tested for antibacterial activity. What saliva component has
antibacterial properties?
A *Lysozyme
B Amylase
C Ceruloplasmin
D Parotin
E Cholesterol

While examining the oral cavity a dentist revealed inflammation of papillae on the border of the
median and posterior third of the back of tongue. What papillae are inflamed?
A *Papillae vallatae
B Papillae fungiformes
C Papillae foliatae
D Papillae filiformes
E Papillae conicae

A 22-year-old female student consulted a physician about fever up to 38oC, weakness, sore
throat. Objectively: there is white coating of the tongue. What histological structures of the
tongue are involved in the formation of this coating?
A *Epithelium of the filiform papillae
B Epithelium of the foliate papillae
C Epithelium of the fungiform papillae
D Epithelium of the circumvallate papillae
E –

Histologic specimen demonstrates an oral cavity organ with mucosa covered with keratinized
stratified squamous epithelium. Specify this organ or its part:
A *Gum
B Inferior surface of tongue
C Labial mucosa
D Uvula
E Soft palate

A histological specimen shows terminal secretory parts of glands made of conic cells with
basophilic cytoplasm and a roundish nucleus in the center. Specify the type of terminal secretory
parts by the type of secretion:
A *Serous
B Sebaceous
C Combined
D Mucous
Е Seromucous

In the process of embryogenesis of dentomaxillary system there was an enamel disturbance


detected. What source of tooth development is damaged in this case?
A *Oral cavity epithelium
B Dental saccule
C Dental bulb
D Mesenchyme
E Mesoderm

In which of the following structures of the oral cavity would taste buds be localized in the
highest concentration?
a. Fungiform papillae
b. Gingiva
c. Filiform papillae
d. Ventral surface of the tongue
e. *Vallatae papillae

The type of epithelium associated with the vermilion zone of the lips is
A stratified squamous nonkeratinized.
B pseudostratified ciliated columnar.
C *stratified squamous keratinized.
D stratified cuboidal.
E stratified columnar.

Passage of a bolus through the esophagus into the stomach is facilitated by which of the
following?
A *Peristaltic activity of the esophageal muscularis externa
B Peristaltic activity of the gastric muscularis mucosae
C Reflux through the pharyngoesophageal sphincter
D Smooth muscle in the esophageal muscularis mucosae
E Reflux through the gastroesophageal sphincter

Digestive system 2

351. Examination of a patient, suffering from atrophic gastritis, revealed megaloblastic anemia.
The anemia is likely to be caused by the deficiency of the following substance:
A *Gastromucoproteid
B Vitamin B6
C Vitamin B1
D Iron
E Erythropoietins

352. It was revealed that a patient with coagulation failure has thrombosis of a branch of inferior
mesenteric artery. What bowel segment is affected?
A *Colon sigmoideum
B Ileum
C Caecum
D Colon transversum
E Colon ascendens

355. The effect of some harmful factors caused focal damage to the gastric epithelium. What
cells are responsible for its regeneration?
A *Cervical mucocytes of glands
B Parietal exocrinocytes of glands
C Principal exocrinocytes of glands
D Endocrinocytes
E Mucocytes of the gland body

356. A patient underwent gastroscopy that revealed insufficient amount of mucus covering the
mucous membrane. This phenomenon is caused by the dysfunction of the following cells of
stomach wall:
A *Cells of prysmatic glandular epithelium
B Parietal cells of gastric glands
C Principal exocrinocytes of gastric glands
D Cervical cells of gastric glands
E Endocrinocytes

372. During histological examination of the stomach it was found out that glands contain very
small amount of pariental cells or they are totally absent. Mucose membrane of what part of the
stomach was studied?
A *Pyloric part
B Fundus of stomach
C Cardiak part
D Body of stomach
E -

373. When the pH level of the stomach lumen decreases to less than 3, the antrum of the stomach
releases peptide that acts in paracrine fashion to inhibit gastrin release. This peptide is:
A *GIF
B Acetylcholine
C Gastrin-releasing peptide (GRP)
D Somatostatin
E Vasoactive intestinal peptide (VIP)

374. Examination of a 43 y.o. patient revealed that his stomach has difficulties with digestion of
protein food. Gastric juice analysis revealed low acidity. Function of which gastric cells is
disturbed in this case?
A *Parietal exocrinocytes
B Main exocrinocytes
C Mucous cells (mucocytes)
D Endocrinous cells
E Cervical mucocytes

375. A 2-year-old child has got intestinal dysbacteriosis, which results in hemorrhagic syndrome.
What is the most likely cause of hemorrhage of the child?
A *Vitamin K insufficiency
B Activation of tissue thromboplastin
C PP hypovitaminosis
D Fibrinogen deficiency
E Hypocalcemia

377. A patient ill with chronic gastritis went for endogastric pH-metry that allowed to reveal
decreased acidity of gastric juice. It is indicative of diminished function of the following cells:
A *Parietal exocrinocytes
B Chief exocrinocytes
C Endocrinocytes
D Cervical cells
E Accessory cells

379. After a gastrectomy, malignant anemia had developed at a patient. Absence of what cells of
stomach glands does cause this pathology?
A *Parietal
B Chief
C Neck mucocytes
D Endocrinocytes
E Goblet cells

380. On histological specimen the submucosa of small intestine is filled with terminal secretory
portion of glands. What part of intestine is presented on specimen?
A * Duodenum
B Colon
C Jejunum
D Ileum
E Appendix

381. On histological specimen of wall of small intestine on the bottom of crypt it was found
groups of cells in apical part of which there are large acidophilic secretory granules; a cytoplasm
is stained basophilic. What cells is it?
A * Paneth cells
B Cells without microvilli
C Endocrine cells
D Goblet cells
E Columnar cells with microvilli

382. In a 1-year-old child, A violation of curdling of maternal milk is observed. What cells of
stomach glands has its activity violated?
A *Chief exocrinocytes
B Parietal exocrinocytes
C Neck mucocytes
D Additional mucocytes
E Exocrinocytes

383. Under influence of harmful factors, there was local damage of the stomach's epithelium.
Due to activity of what cells did the regeneration occur?
A* Neck mucocytes
B Parietal exocrinocytes
C Chief exocrinocytes
D Endocrinocytes
E Mucocytes of glands body

384. At endoscopic research of stomach it was observed damage of epithelial lining of mucus
layer. Due to activity of what glandulocytes, is reparative regeneration possible…or what do you
think would cause the repairs of this damaged layer?
A *Undifferentiated neck mucocytes
B Additional mucocytes
C Chief exocrinocytes
D Parietal exocrinocytes
E Covering glandular epithelium

385. To the patient with chronic gastritis, the pH measurement was done. A decreased acidity of
gastric juice was revealed. Function of which cells was diminished?
A * Parietal exocrinocytes
B Chief exocrinocytes
C Endocrinocytes
D Neck mucocytes
E Additional cells

386. On an electronic photomicrograph of the fragment of stomach gland the large cell of
irregular spherical shape was observed. In its cytoplasm there is a high amount of intracellular
channels and mitochondria. Define this cell.
A * Parietal
B Chief
C Undifferentiated
D Mucous
E Endocrine

387. At histological investigation of biopsy material of the stomach of a patient, A substantial


diminishing or complete absence of parietal cells in glands was seen. The mucus envelope of
what part of stomach was investigated?
A * Pyloric
B Fundus
C Cardiac
D Body
E-

388. At an inspection of the patient of 43 it was revealed that in his stomach proteins are badly
digested. The analysis of gastric juice found out low acidity. The function of what cells of
stomach is broken in this case?
A * Parietal exocrinocytes
B Chief exocrinocytes
C Mucocytes
D Endocrine cells
E Neck mucocytes

A patient ill with chronic gastritis went for endogastric pH-metry that allowed to reveal
decreased acidity of gastric juice. It is indicative of diminished function of the following cells:
A Parietal exocrinocytes
B Chief exocrinocytes
C Endocrinocytes
D Cervical cells
E Accessory cells
390. On the histological specimen of fundus of stomach, comparatively large cells with an
acidophilic cytoplasm were noticed in its composition. Electron microscopy of these cells shows
the presence of complicated system of intracellular channels. What component of gastric juice is
produced by these cells?
A * Hydrochloric acid
B Pepsinogen
C Mucus
D Serotonin
E Gastrin

Secretion of what gastrointestinal hormones will be primarily decreased as a result of duodenum


removal?
A *Cholecystokinin and secretin
B Gastrin
C Histamine
D Gastrin and histamine
E Neurotensin.

A patient ill with chronic gastritis went for endogastric pH-metry that allowed to reveal
decreased acidity of gastric juice. It is indicative of diminished function of the following cells:
A *Parietal exocrinocytes
B Chief exocrinocytes
C Endocrinocytes
D Cervical cells
E Accessory cells

A patient has a disturbed absorption of fat hydrolysates. It might have been caused by a deficit in
the small intestine cavity:
A *Of bile acids
B Of bile pigments
C Of lipolytic enzymes
D Of sodium ions
E Of liposoluble vitamins

An electron microphotography of a fragment of proper gastric gland shows a big irregular round-
shaped cell. There are a lot of intracellular tubules and mitochondria in the cytoplasm. Specify
this cell:
A *Parietal cell
B Principal cell
C Undifferentiated cell
D Mucous cell
E Endocrine cell

A 35-year-old man with peptic ulcer disease has undergone antrectomy. After the surgery
secretion of the following gastrointestinal hormone will be disrupted the most:
A *Gastrin
B Histamine
C Secretin
D Cholecystokinin
E Neurotensin

A patient complaining of heartburn has undergone biopsy of the gastric mucosa. In the sample
there are numerous cells with oxyphilic cytoplasm in the glandular epithelium. Name these cells:
A. *Exocrine parietal cells
B. Exocrine chief cells
C. Mucous cells
D. Epithelial cells
E. Endocrine cells
Digestive system 3

357. Proliferation of connective tissue in the parenchyma of liver (fibrosis) caused by chronic
diseases is typically accompanied by an impairment of blood circulation in the classic lobules.
What is the direction of blood flow in these lobules?
A * From the periphery to the center
B From the center to the periphery
C Around the lobule
D From the top to the base
E From the base to the top

376. A viral infection has damaged cells that form walls of bile capillaries. This stimulated
conditions for inflow of bile into the blood of sinusoidal capillaries. What cells are damaged?
A *Hepatocytes
B Kupffer's cells
C Ito cells
D Pit-cells
E Endotheliocytes

391. Since 14 years the patient has been suffering from diabetes mellitus. What cells of pancreas
do not function?
A * B-cells
B A-cells
C D-cells
D D1-cell
E PP-cells

395. As a result of viral infection cells of the walls of bilious capillaries were damaged. It created
conditions for entering of bile to the blood of sinusoidal capillaries. What cells were damaged?
A* Hepatocytes
B Kupffer cells
C Ito cells
D Pit-cells E
Endotheliocytes

396. At ultramicroscopical investigation of population of "dark" hepatocytes in the cytoplasm of


cells it was noticed the well-developed granular endoplasmic reticulum. What function does this
organelle perform in this cells?
A *Synthesis of albumens of blood plasma
B Synthesis of carbohydrates
C Desintoxication
D Production of bile
E Depositing of calcium ions

397. At histological specimen the parenchyma of organ is presented by lobules, which have a
hexagonal prisms and consist of anastomosing plates with sinusoidal capillaries between them.
These capillaries radially diverge from central vein. What anatomic organ has such
morphological structure?
A *Liver
B Pancreas
C Thymus
D Spleen
E Lymphatic node
398. In blood of patient, a low level of albumins and fibrinogen was observed. Decline activity
of what organelles of hepatocytes did cause this phenomenon?
A *Granular endoplasmic reticulum
B Agranular endoplasmic reticulum
C Mitochondria
D Complex Golgi
E Lysosomes

399. In the ration of man there is a plenty of carbohydrates. What structures will appear in the
cytoplasm of hepatocytes?
A *Glycogen granules
B Drops of lipid
C One large fatty droplet
D Increase of amount of free ribosomes
E Inclusions of lipofuscin

400. A patient of 55 years, is observed by endocrinologist concerning violation of endocrine


functions of pancreas which results in a decrease of the level of hormone glucagon in blood. The
function of what cells of this gland is broken in this case?
A * A-cells of Langerhans islets
B B- cells of Langerhans islets
C D- cells of Langerhans islets
D D1- cells of Langerhans islets
E PP- cells of Langerhans islets

401. During secretory cycle of pancreatic cell in the apical part of it's cytoplasm granules of
secretions appear and disappear. To what structural elements is it possible to include these
granules?
A * Inclusions
B Microphilaments
C Lysosomes
D Excretory vacuoles
E Granular endoplasmic reticulum

Examination of a patient suffering from chronic hepatitis revealed a significant decrease in the
synthesis and secretion of bile acids. What process will be mainly disturbed in the patient’s
bowels?
A *Fat emulsification
B Protein digestion
C Carbohydrate digestion
D Glycerin absorption
E Amino acid absorption

A patient consumed a lot of reach in proteins food that caused increase of rate of proteolytic
enzymes of pancreatic juice. It is also accompanied by increase of rate of the following enzyme:
A *Trypsin
B Pepsin
C Enterokinase
D Gastrin
E Renin

A 30-year-old woman has subnormal concentration of enzymes in the pancreatic juice. This
might be caused by the hyposecretion of the following gastrointestinal hormone:
A *Cholecystokinin-pancreozymin
B Somatostatin
C Secretin
D Gastro-inhibiting peptide
E Vaso-intestinal peptide

Before the cells can utilize the glucose, it is first transported from the extracellular space through
the plasmatic membrane inside them. This process is stimulated by the following hormone:
A *Insulin
B Glucagon
C Thyroxin
D Aldosterone
E Adrenalin

A specimen of a parenchymal organ shows poorly delineated hexagonal lobules surrounding a


central vein, and the interlobular connective tissue contains embedded triads (an artery, a vein
and an excretory duct). What organ is it?
A *Liver
B Pancreas
C Thymus
D Spleen
E Thyroid

In what organ biotransformation (metabolic transformation) of most medicinal agents occurs


upon their introduction into an organism?
A *Liver
B Kidneys
C Intestine
D Skin
E Lungs

In a liver biopsy from a long-time drug user which of the following hepatocyte organelles would
be expected to be more extensive than normal?
a. Rough endoplasmic reticulum
b. Golgi apparatus
c. Lysosomes
d. Peroxisomes
e. *Smooth endoplasmic reticulum

Which description is true of pancreatic zymogens?


a. Are packaged for secretion in the SER
b. Are synthesized on free ribosomes
c. *Are inactive until they reach the duodenal lumen
d. Are stored in the basal cytoplasm of acinar cells
e. Are produced by cuboidal cells lining the pancreatic duct

Which feature is unique to the exocrine pancreas?


a. Insulin-secreting β cells
b.* Centroacinar cells
c. Predominately serous secretory cells
d. Striated interlobular ducts
e. Striated interlobular ducts

Which description is true of the bile canaliculi?


a. Are bordered directly by endothelial cells
b. Are part of the portal triad
c. Are surrounded by the hepatic sinusoids
d. *Lumens are entirely sealed by junctional complexes
e. Normally contain some blood plasma

Which description is true of the gallbladder?


a. Dilutes bile
b. Absorbs bile
c. *Secretes mucus
d. Has a thick submucosa
e. Is covered entirely by serosa

Which description is true for the hepatic space of Disse?


a. Is surrounded by the hepatic sinusoid
b. Contents flow toward the central vein
c. *Is directly contacted by hepatocytes
d. Lumen sealed by junctional complexes
e. Contents empty into canals of Hering lined by cholangiocytes

A 50-year-old woman presents to the family medicine clinic. She admits to drinking a six-pack
of beer each day with a little more intake on weekends. Laboratory tests show elevated alanine
aminotransferase/serum glutamic oxaloacetic transaminase (AST/SGOT). Her sclerae appear
jaundiced and her serum bilirubin is 2.5 mg/dL (normal 0.3-1.9 mg/dL). A biopsy shows hepatic
fibrosis with significant loss of normal lobular structure. Jaundice is most likely to result when
the proper location or orientation of what hepatic structures is disrupted?
a. Central veins
b. Spaces of Disse
c. Kupffer cells
d. *Hepatocytes
e. Merging sinusoids

A young child presents with hepatomegaly and renomegaly, failure to thrive, stunted growth, and
hypoglycemia. A deficiency in glucose-6-phosphatase is identified and the diagnosis of von
Gierke
disease is made. What cellular structures would be expected to accumulate in hepatocytes during
progression of this disorder?
a. Chylomicrons
b. *Glycogen granules
c. Mitochondria
d. Zymogen granules
e. Ribosomes

Respiratory system

402. Study of a tubular organ revealed that its median membrane consists of solid hyaline rings.
What epithelium lines mucous membrane of this organ?
A *Multinuclear columnar ciliated
B Monostratal columnar glandular
C Monostratal columnar with a border
D Multistratal squamous nonkeratinizing
E Monostratal cubical

403. A patient was admitted to the hospital with an asphyxia attack provoked by a spasm of
smooth muscles of the respiratory tracts. This attack was mainly caused by alterations in the
following parts of the airways:
A *Small bronchi
B Median bronchi
C Large bronchi
D Terminal bronchioles
E Respiratory part

404. A patient with an acute rhinitis has hyperemia and excessive mucus formation in nasal
cavity. What epithelial cells of mucous membrane have the intensified activity?
A *Goblet cells
B Ciliated cells
C Microvillous cells
D Basal cells
E Endocrine cells

405. Premature infants have syndrome of respiratory failure. Failure of what aerohematic barrier
component underlies this pathology?
A *Surfactant
B Capillary endothelium
C Basal membrane of endothelium
D Basal membrane of alveolocytes
E Alveolocytes

407. Lung of premature infant is presented on electronic photomicrography of biopsy material.


Collapse of the alveolar wall caused by the deficiency of surfactant was revealed. Dysfunction of
what cells of the alveolar wall caused it?
A *Alveocytes type II
B Alveocytes type I
C Alveolar macrophages
D Secretory cells
E Fibroblasts

409. A patient with thrombophlebitis of lower extremities had got chest pains, blood spitting,
growing respiratory failure that caused his death. Autopsy revealed multiple pulmonary
infarctions. What is the most probable reason of their development?
A *Pulmonary artery embolism
B Pulmonary artery thrombosis
C Bronchial artery thrombosis
D Bronchial artery embolism
E Pulmonary venous thrombosis

410. Electronic microphotography of pulmonary alveoli’s wall presents a big cell. Its cytoplasm
has a lot of mitochondria, developed Golgi apparatus, osmiophilic lamellate corpuscles. What is
the main function of this cell?
A *It produces surfactant
B It is a component of blood-air barrier
C It warms the air
D It purifies the air
E It absorbs microorganisms

411. A pathological process in bronchi resulted in epithelium desquamation. What cells will
regenerate bronchial epithelium?
A *Basal
B Intercalary
C Ciliate
D Endocrinal
E Goblet

412. On the electronic photomicrograph of the wall of lung's alveoli, a large cell is present, in the
cytoplasm of this cell there are much mitochondria, well-developed complex Golgi, osmiophilic
lamellar bodies. What is the basic function of this cell?
A * Production of surfactant
B It is the component of aero-hematic barrier
C Warming the air
D Clearing the air
E Capturing microorganisms

413. On a histological specimen of the lungs, it was noticed that the structure of the wall consists
of Simple cuboidal ciliary epithelium. The muscular plate consists of smooth muscle cells and
fold mucus is absent. What structure is it?
A *Terminal bronchus
B Small bronchus
C Middle bronchus
D Large bronchus
E Main bronchus

414. On a specimen of one of departments of the respiratory system it was found a tubular organ,
in which simple cuboidal epithelium is determined, a well-developed muscular envelope, glands
and cartilage is absent. Name this organ.
A * Small bronchus.
B Trachea
C Larynx
D Large bronchus
E Middle bronchus

415. On a microscopic specimen of the lungs of man ill with the pneumonia (inflammation of
lungs tissue), it is noticed that there are damaged cells, which are responsible for a respiratory
function. What cells of the alveolar wall are they?
A * Type I alveolocytes
B Type II alveolocytes
C Macrophages
D Clara cells
E Lymphocytes

416. On a specimen a hollow organ is presented. A mucous membrane is covered by two-layered


ciliary epithelium which shifts to one-layered. The muscular plate of mucus envelope is well
developed in relation to the thickness of all other parts of entire wall, also cartilage and glands
are absent. What organ is presented in specimen?
A * Small bronchus
B Middle bronchus
C Trachea
D Larynx
E Urinary bladder

417. A patient was hospitalized with the attack of suffocation (difficulty in breathing), caused by
spasm of smooth muscles of the respiratory tract. Name the departments of auriferous ways,
which this attack is connected with?
A * Small-caliber bronchi
B Middle-caliber bronchi
C Large-caliber bronchi
D Terminal bronchioles
E Respiratory department

418. At the autopsy of a dead man of 65 years, who suffered the disease of lungs, a pathological
process was seen, mainly localized in the bronchi. The glands, cartilaginous islands and
pseudostratified ciliary epithelium were expressly visible at histological research. Name these
bronchi?
A * Middle bronchus
B Main bronchial tubes
C Large bronchus
D Small bronchus
E Terminal bronchioles

419. On an electronic photomicrograph of the cells of alveoli, cells that take part in the formation
of aero-hematic barrier are seen. Which cells are these?
A * Type I alveolocyte
B Type II alveolocyte
C Alveolar macrophages
D Clara cells
E Epitheliocytes with microvilli

420. At the worker of chemical manufacture after inspiration of a caustic pair death of part of
ciliary epitheliocytes of bronchi was observed. What cells will take place in regeneration of this
epithelium?
A *Basal cells
B Goblet cells
C Endocrine cells
D Ciliary cells
E Cells without cilia

421. On an electronic photomicrograph, it is observed structures, presented as opened sacs, lined


from the internal surface with simple epithelium, which consists of respiratory and secretory
cells. Which structures are these?
A *Alveoli
B Bronchioles
C Acini
D Small bronchus
E Terminal bronchioles

422. In the epithelium of auriferous ways there are cells with a dome-shaped apical part on their
surface of which microvilli are situated. In this cell it is also observed well developed synthetic
apparatus, and in the apical part - secretory granules. Name this cell.
A * Clara (club) cells
B Goblet cells
C Endocrine
D A cell is without microvilli
E Cambial cell

423. The syndrome of respiratory insufficiency develops at prematurely born children.


Insufficiency of which component of the aero-hematic barrier causes this pathology? A *
Surfactant
B Endothelium of capillaries
C Basal membrane of endothelium
D Basal membrane of alveolocytes
E Alveolocytes

424. On a histological specimen of trachea a pseudostratified ciliary epithelium is observed with


comparatively low cells of oval or three-cornered form. They do not have the apical surface of
epithelium. In several cells the process of mitosis is visible. What is the function of these cells?
A *Regeneration
B Represent a part of mucociliary complex
C Secrete mucus
D Secrete a surfactant
E Secrete biologically active substances

425. As a result of pathological process there is a desquamation in the bronchial epithelium.


What cells could cause regeneration?
A * Basal
B Inserted
C Ciliary
D Endocrine
E Goblet
426. In a child of two years the evacuation of mucus from bronchi is disturbed. With violation of
functions of which organelles of the cells of the bronchial epithelium can it be connected with?
A *Cilia
B Mitochondria
C Cytoplasmic reticulum
D Microvilli
E Lysosomes

427. As a result of trauma of a man's nose of 32years of age. The mucus layer of superior nasal
concha was damaged. What consequences did it result in?
A *Violation of smell sensation
B Insufficient warming of air
C Insufficient moistening of air
D Insufficient warming and moistening of air
E Violation of cleaning of air

188. Alveolar space of the acinus was invaded by some bacteria which interacted with the
surfactant. This led to the activation of the cells that are localized in the alveolar walls and on the
surface. What cells are these?
A *Alveolar macrophages
B Alveolocytes type I
C Endothelial cells
D Clara cells
E Alveolocytes type II

After breathing with poisonous steams there is an increased quantity of slime in respiratory
passages of a chemical production worker. What of respiratory tract epithelial cells participate in
mucousa moistening?
A *Goblet cells
B Fibroblasts
C Endocrine cells
D Langerhans cells
E Intercalated cells

Work in a mine is known to cause inhalation of large amounts of coal dust. Inhaled coal dust can
be detected in the following pulmonary cells:
A *Alveolar macrophages
B Respiratory epithelial cells
C Secretory epithelial cells
D Capillary endothelial cells
E Pericapillary cells

A microslide of the lung tissue sample taken from a patient with pneumonia shows damage to the
cells that carry out respiratory function. What cells of the alveolar wall are damaged?
A. *Type 2 alveolar cells
B. Type1 alveolar cells
C. Macrophages
D. Club cells
E. Lymphocytes

Urinary system

428. Examination of mountain climbers who have spent a long time in a high-altitude region
revealed increase of erythrocyte number (over 610^12/l) and hemoglobin concentration (over
170 g/l). What mechanism caused this phenomenon?
A *Intensified production of erythropoietin by the kidneys
B Weakening of erythrocyte hemolysis in bloodstream
C Improved ability of tissue for oxygen utilization
D Intensified processes of anoxic energy production
E Weakening of intracellular erythrocyte hemolysis

429. Electron-microscope investigation of cortical substance of a kidney reveals some structures


lined with columnar epithelium that normally has brush border and deep plicae of plasmolemma
in its basal part. There is a big number of mitochondrions between these plicae. These structures
belong to the following part of a nephron:
A *Proximal tubule
B Distal convoluted tubule
C Henle's loop
D Renal corpuscle
E Distal straight tubule

230. A microphotography represents a fragment of cortical substance of a kidney. This fragment


contains thick spot cells and juxtaglomerular cells with big secretory granules. What kidney
structure is represented?
A *Juxtaglomerular apparatus
B Renal corpuscle
C Filtering barrier
D Prostaglandin apparatus
E Choroid glomus

432. A patient suffering from chronic renal insufficiency has got osteoporosis. Osteoporosis was
caused by abnormal synthesis of the following regulator of mineral metabolism in kidneys:
A *1,25(OH)2 D3 formation
B Proline hydroxylation
C Lysine hydroxylation
D Glutamate carboxylation
E Cortisol hydroxylation

433. In a histological specimen of adrenal cortex there are petite polygonal cells that form
roundish clusters and contain some lipidic inclusions. What part of adrenal is presented in this
histological specimen?
A *Glomerular zone
B Intermedial zone
C Fasciolar zone
D Reticular zone
E -

434. Electron micrograph of a kidney fragment presents an afferent arteriole with big cells under
endothelium. These cells contain secretory granules. Name this type of cells:
A *Juxtaglomerular
B Mesangial
C Smooth muscular
D Juxtavascular
E Interstitial

435. The low specific gravity of the secondary urine (1002) was found out in the sick person.
Wat I the most distant part of nephron where concentration of secondary urine takes place?
A *In the collecting duck
B In the nephron's glomerulus
C In proximal tubule of nephron
D In ascending part of loop of Henle
E In distal tubule of nephron

436. A patient has been given high doses of hydrocortisone for a long time. This caused atrophy
of one of the adrenal cortex zones. Which zone is it?
A *Fascial
B Glomerular
C Reticular
D Glomerular and reticular
E-

437. A histological specimen of a kidney shows a part of the distal tubule going between the
afferent and efferent arteriole. The cells building the tubule wall have dense nuclei; basal
membrane is absent. Such structural formation is called:
A *Macula densa
B Juxtaglomerular cells
C Mesangial cells
D Juxtavascular cells
E -

438. A histological specimen of kidney shows a structure consisting of a glomerulus of


fenestrated capillaries and a bilayer epithelial capsule. Specify this structure:
A *Renal corpuscle
B Proximal tubule
C Distal tubule
D Henle's loop
E Receiving tube

439. At the electronic microscopy of kidney tubules which were covered by cuboidal epithelium
were observed. In an epithelium it could be distinguished light and dark cells. There are little
organelles in light cells. A cytoplasmic membrane forms folds. These cells provide the
reabsorption of water from primary urine into the blood. Dark cells structurally and functionally
resemble parietal cells of the stomach. What tubules are presented on electronogram?
A *Collecting duct
B Proximal convolutes tubules
C Distal convoluted tubules
D Ascending part of loop of Henle
E Descending part of loop of Henle

440. During a clinical inspection at 35-years-old woman with the disease of kidneys in her urine
was found blood cells, fibrinogen, that it is certainly related to violation of kidney filter. What
structures does this filter consist of?
A *Endothelium of glomerular capillaries, three-layered basal membrane, podocytes B Three-
layered basal membrane
C Endothelium of glomerular capillaries, basal membrane
D Podocytes, basal membrane
E Endothelium, podocytes

441. In an experimental model using a rat it morphological damage of epithelial cells of distal
parts of nephron was performed. What functional processes in kidney will suffer?
A * Reabsorption of ions and water
B Reabsorption of glucose
C Reabsorption of sodium and glucose
D Reabsorption of proteins
E Filtration

442. In a patient with suspicion of glomerulonephritis, in the secondary urine it was found the
presence of proteins Albumin /albuminuria/ and glucose/glucosuria during two weeks. The
function of what part of kidney was broken?
A * Proximal convolutes tubules
B Distal convoluted tubules
C Loop of Henle
D Collecting ducts
E Juxtaglomerular apparatus
443. An important component of kidney filtration barrier is the three-layered basal membrane
which has the special reticulated structure of its middle electron-dense layer. Where is this basal
membrane located?
A *In kidney glomeruli
B In the capillaries of peritubular capillary network
C In the proximal convolutes tubules
D In the loop of Henle
E In the distal convoluted tubules

444. During experiment in an animal with the help of narrowing of kidney artery permanent
increase of blood pressure was obtained. The function of what cells of kidney does cause this
effect?
A * Juxtaglomerular cells
B Podocytes
C Endotheliocytes
D Interstitial cells
E Cells of macula densa (dense spot)

445. In a patient (27 years) at urine analysis it was found remnants of proteins and glucose. What
part of nephron is damaged? A * Proximal convolutes tubules
B *Ascending part of loop of Henle
C Descending part of loop of Henle
D Distal convoluted tubules
E Kidney glomerus

446. On histological specimen of kidney cells of distal convoluted tubules which densely adjoin
to the kidney glomerus were observed. Their basal membrane is very thin and does not form
folds. These cells perceive the changes of sodium concentration in urine and influence on the
secretion of rennin by juxtaglomerular cells. What cells are these?
A * Cells of macula densa (dense spot)
B Juxtaglomerular cells
C Mesangial cells
D Podocytes
E Endothelium of glomerular capillaries

447. A patient after exogenous intoxication had a threat of development of ascending infection of
urinary tract because of the loss of acidic reaction of urine. What cells were damaged in kidney?
A * Dark cells of collective ducts
B Light cells of collective ducts
C Epitheliocytes of proximal convolutes tubules
D Flat cells of loop of Henle
E Epitheliocytes of distal convoluted tubules

448. On an electron photomicrograph the fragment of kidney afferent arteriole is presented,


under the endothelium of this vessel, there are seen large cells with secretory granules. Indicate
the name of this cells.
A * Juxtaglomerular cells
B Mesangial cells
C Smooth muscle cells
D Juxtavascular cells
E Interstitial

449. As a norm at laboratory investigation of urine blood cells does not appear in it. What
structure of nephron prevents passage of formed blood elements into the primary urine?
A *Basal membrane of glomerular capillaries
B Juxtavascular cells
C Mesangial cells
D Epithelium of external wall of the capsule of glomerus
E Epithelium of loop of Henle

450. On electron photomicrograph one of the reions of nephron cells of cuboidal shape are
determined, the apical surface of this cells contains microvilli, and basal - basal striations with
mitochondria, located between invaginations of cytolemma. Indicate the part of the nephron.
A * Proximal convolutes tubules
B Collecting ducts
C Distal convolutes tubules
D Flat cells of loop of Henle
E Capsule of glomerus

451. At histological investigation of kidney in its cortex a tubule, which is lined by simple
сuboidal epithelium with microvilli was observed. It's cytoplasm was stained basophilic.
Indicate, what segment of nephron was observed on the specimen.
A * Proximal convolutes tubules
B Collecting ducts
C Distal convolutes tubules
D Distal straight tubules
E Loop of Henle

452. On the electronic photomicrograph of fragment of kidney glomerus a large epithelial cell
with large and small processes. These processes anchor to the basal membrane of capillaries.
Indicate this cell:
A *Podocyte
B Juxtavascular cell
C Smooth muscle cell
D Endotheliocyte
E Mesangial cells

453. In histological specimen of kidney the part of distal convoluted tubule, which is situated
between afferent and efferent arteriole is seen. In the cells which represent the wall a tubule, the
nucleus is dark, a basal membrane is absent. What is the name of such structural composition?
A *Dense spot
B Endotheliocytes
C Mesangial cells
D Juxtavascular cells
E Juxtaglomerular cells

470. The impact of oxytocin on uterus wall helps to stop uterine bleeding after labor. What
membrane of this organ reacts on the effect of this hormone?
A *Myometrium
B Endometrium
C Perimetrium
D Parametrium
E Submucous membrane

Due to the use of poor-quality measles vaccine for preventive vaccination, a 1- year-old child
developed an autoimmune renal injury. The urine was found to contain macromolecular proteins.
What process of urine formation was disturbed?
A * Filtration
B Reabsorption
C Secretion
D Reabsorption and secretion
E Secretion and filtration

Electron micrograph of the kidney shows fenestrated endothelium lying on the basement
membrane; the external surface of the membrane has adjacent dendritic epithelial cells. What do
these structures form in the kidney?
A Filtration barrier
B Juxtaglomerular apparatus
C Distal nephron
D Henle’s loop
E Proximal nephron

An electron micrograph of a nephron segment shows cuboidal cells with ciliated lining on their
apical surfaces; their basal surface shave basal striation with mitochondria located between the
cytolemma invaginations. Name the described nephron segment:
A. Proximal tubule
B. Collecting ducts
C. Distal tubule
D. Thin limbs of Henle’s loop
E. Glomerular capsule

Male reproductive system

454. A male patient underwent an operation on account of inguinal hernia. During the operation a
surgeon damaged content of the inguinal canal. What structure was damaged?
A *Funiculus spermaticus
B Urarchus
C Lig. teres uteri
D Lig. inguinalе
E -

455. The structure of seminiferous tubules was damaged as a result of mechanical trauma of the
testis. Define the result of this pathology.
А. *Aspermatogenesis.
В. Polyspermy.
С. Spermatogenesis will not change.
D. Monospermy.
E. Synthesis of testosterone will decrease.

456. Large columnar epithelial cells of seminifireous tubule are seen in histologic specimen of
testis. Their basis rest upon basal membrane and apical pole extend in lumen of the tubule. Name
these cells.
А. *Sustentocytes.
В. Leydig cells.
С. Spermatogonium.
D. Spermatidys
E. Spermatozoa.

457. Numerous seminiferous tubules are seen in histologic specimen. The aggregation of the
polygonal shape cells with big amount of lipid inclusions are situated between tubules in loose
connective tissue. Name these cells.
А. Sustentocytes.
В. * Leydig cells.
С. Decidual cells.
D. Follicular cells.
E. Sertoli cells.

458. The transverse section of tubules of the male reproductive system are seen in histologic
specimen. The groups of high ciliated cells and low cuboidal cells are situated in tubular
epithelium. Define the part of the male reproductive system.
А. Seminiferous tubules.
В. Straight tubules.
С. Ductus deferens.
D. Ejaculatory duct.
E. *Ductus epididymis.

459. Endoсrine examination has shown that level of testosterone in blood plasma of men
increased. Name the cells which produce this hormone.
А. Sustentocytes.
В. Cells of anterior hypothalamus nucleuses.
С. Sertoli cells.
D. * Leydig cells.
E. Spermatogenic cells.

460. Muscular-glandular organ having lobulated construction is seen in histologic specimen. The
lobules have a separate glands with excretory ducts which are opened in channel located in the
centre of the organ. The wall of the channel is covered by transitional epithelium. Name this
organ.
А. Testis.
В. *Postate gland.
С. Seminal vesicle.
D. Epididymis.
E. Mammary gland.

461. Glandular cells of testis were ruined experimentally. What changes possible in blood?
А. *Reduction level of testosterone.
В. Reduction amount of androgen-binding protein.
С. Reduction level of lutropin.
D. Absence of the changes.
E. Reduction of the gonadoliberin synthesis.

462. In the pubertal period cells of the male sexual glands start producing the male sexual
hormone testosterone that is responsible for formation of the secondary sexual characters. What
cells of the male sexual glands produce this hormone?
A Leydig cells
B Sustenocytes
C Sertoli’s cells
D Sustentacular cells
E Spermatozoa

463. In one of the phases of spermatogenesis, there are changes of nuclei and cytoplasm of
spermatids, which results in the formation of mature gametes. Name the phase of gametogenesis.
A *Formation
B Maturating
C Growth
D Multiplication
E Proliferation

464. A couple complains about difficulty in conceiving a child. After an inspection, it was
discovered - the spermatogenous epithelium of the testicle had been damaged, which resulted in
absence of spermatozoa in the seminal fluid and as a result - infertility. What part of the testicles
had suffered?
A * Convoluted seminiferous tubules
B. Straight seminiferous tubules
C. Rete testis (testicular network)
D. Epididymis
E. Collecting tubules

465. During the maturation of the cells of the male sexual glands, the glands begin to produce the
male sex hormone testosterone, which causes the appearance of the secondary sexual signs.
What cells of male sexual glands do produce this hormone?
A * Leydig cells
B Sustentocytes
C Sertoli cells
D Supporting cells
E Spermatozoa

466. As a result of orchitis (inflammation of testicles) in a man of 43, the production of


spermatozoa has been violated. In what part of testicles does this pathological change take place?
A * Convoluted seminiferous tubules
B Rete testis
C Straight seminiferous tubules
D Collecting tubules of testicles
E Ductus of epididymis

467. On a histological specimen it is seen the transverse cut of tubules, whose wall of which
consists of mucus, muscle and adventitial envelopes. The epithelium of the mucus is presented
with high columnar cells with stereocilia and low cambial cells. What organ of the male sexual
system was investigated?
A * Ductus epididymis.
B Testicle.
C Prostatic gland.
D Ejaculatory duct.
E Urethra.

468. On a histological specimen we can see an organ which is covered from the outside by
serosal and fibrous (albugenuos) envelopes. The stroma of this organ is made up of loose
connective tissue with Leydig cells on the inside. Also, the parenchyma is presented by tubules
and the internal surface of tubules lined with spermatogenous epithelium. What organ is it?
A *Testicle
B Epididymis
C Prostatic gland
D Mammary gland
E Ovary

A patient complains of frequent and difficult urination. Imperfection of what formation can cause
it?
A *Prostate
B Testicles
C Bulb-uretic glands
D Testicle adnexa
E Sperm bubbles

Testosterone and its analogs increase the mass of skeletal muscles that allows to use them for
treatment of dystrophy. Due to interaction of the hormone with what cell substrate is this action
caused?
A *Nuclear receptor
B Membrane receptors
C Ribosomes
D Chromatin
E Proteins-activators of transcription

Parents of a 10-year-old boy consulted a doctor about extension of hair-covering, growth of


beard and moustache, low voice. Intensified secretion of which hormone must be assumed?
A *Of testosterone
B Of somatotropin
C Of estrogen
D Of progesterone
E Of cortisol

A 30-year-old female exhibits signs of virilism (growth of body hair, balding temples, menstrual
disorders). This condition can be caused by the overproduction of the following hormone:
A *Testosterone
B Oestriol
C Relaxin
D Oxytocin
E Prolactin

It is necessary to perform urinary bladder catheterization of an adult man. Resistance to the


catheter can occur in the following structure or part of the urethra:
A *Membranous part
B Prostatic part
C Spongiose part
D External urethral orifice
E Internal urethral orifice

A specimen presents an endocrine system organ covered with capsule made of connective tissue.
Septa branch off from the capsule inwards and divide the organ into lobules. Each lobule consists
of two cell types: neurosecretory pinealocytes (polygonal cells with processes) located in the
center and gliacytes (astrocytes) located at the periphery. What organ is represented in this
specimen?
A *Epiphysis
B Pituitary gland
C Hypothalamus
D Thyroid gland
E Adrenal medulla

Electron diffraction pattern of a spermatozoon clearly presents a sheath like structure localized in
the spermiocalyptrotheca and participating in dissolution of ovular membranes. Name this
structure:
A *Acrosome
B Centriole
C Microtubules
D Axoneme
E Segmented columns

Female reproductive system

470. The impact of oxytocin on uterus wall helps to stop uterine bleeding after labor. What
membrane of this organ reacts on the effect of this hormone?
A *Myometrium
B Endometrium
C Perimetrium
D Parametrium
E Submucous membrane

478. In sick women as a result of inflammation, the endocrine function of follicle cells of the
ovary is broken. Synthesis of which hormones would be affected?
A *Estrogens
B Progesterone
C Lutropin
D Follicle stimulating hormone
E Folistatin

479. A woman 25 years, a month after giving birth complained to the doctor about the decline in
formation of milk. The lack of what hormone resulted in such state?
A* Prolactin
B Somatostatin
C Adrenocorticotropic hormone
D Insulin
E Glucagon

480. On a specimen of the ovary, stained with hematoxylin-eosin stain, it was observed a follicle
in which the cells of the follicular epithelium are arranged in 1-2 layers having a cuboidal shape
and around the oocyte a bright red envelope is prominent. Name this follicle.
A *Primary
B Primordial
C Secondary
D Mature
E Athretic

481. A patient of 35 years, is diagnosed of infertility. In the gynecological department, a


diagnostic biopsy of the endometrium is done. After the microscopic investigation, it was
determined that a mucus envelope is edematous, uterine glands are convoluted and filled with a
thick secretion. What hormone causes such changes in endometrium?
A. *Progesterone
B. Estrogen
C. Testosterone
D. Somatotropine
E. Adrenocorticotropic hormone

482. Stopping of bleeding after births is connected with the influence of hormones on the
structures of uterus. What component of the uterine wall plays the main role in this process?
A *Middle layer of the myometrium
B Endometrium
C Internal layer of the myometrium
D Superficial layer of the myometrium
E Perimetrium

483. Stopping of bleeding after births is connected with the influence of oxytocin on the wall of
uterus. What envelope of this organ reacts as a result of the action of this hormone?
A * Myometrium
B Endometrium
C Perimetrium
D Parametrium
E Submucous

484. A woman has hyperemia of ovary, increase of permeability of hemato-follicle barrier with
the progressive development of edema, infiltration of the follicle wall by leucocytes. The volume
of follicle is enlarged and the wall of the follicle is thinned. What period of ovarian cycle is
described?
A *Preovulatory stage
B Ovulation
C Menstrual period
D Postmenstrual period
E Rest period

485. A woman of 25 years, a month after giving birth complained to a doctor about a decline in
the formation of milk. The lack of what hormone resulted in such state?
A * Prolactin
B Somatostatin
C Adrenocorticotropic hormone
D Insulin
E Glucagon
486. At the microscopic research on a biopsy material of the endometrium of a woman, who
suffers infertility, it was found out that there were changes in its structure, caused by the action
of hormone progesterone. Where is this hormone produced?
A *Yellow body of ovary
B In the follicles of the ovary
C In the anterior hypophysis
D In the posterior hypophysis
E In the hypothalamus

487. A histological picture of the endometrium has the following characteristic signs: edema,
presence of convoluted glands with widened lumen (which secrete plenty of mucus), no mitotic
cells, and decidual cells in the stroma. What stage of ovarian cycle is described?
A*Secretory (premenstrual).
B Menstrual
C Regeneration
D Proliferation
E Rest

488. In a woman of 40 years a weak childbirth occurs, as a result of the weakness of contractile
ability of the myometrium. To help this situation, what hormone needs to be injected?
A *Oxytocin
B Hydrocortizone
C Dexametazone
D Aldosterone
E Prednisolon

489. A patient with adenoma of the hypophysis (tumor of anterior hypophysis), there is an
increase of the duration in the phase of growth of follicles. What is the duration of growth period
of oocytes during normal oogenesis?
A *12-14 days
B A few tens of years (from 10-13 to 40-50) after birth.
C After birth and to the end of sexual maturity.
D From 3 months of prenatal development to birth
E 28 days

490. A cesarean section operation was performed on a patient, in which a considerable length of
the wall of the uterus was cut and the fetus was taken out. What is the mechanism of healing of
the sewn myometrium?
A *Formation of connective tissue scar
B Formation of new smooth muscle tissue
C Formation of striated muscle fibers
D Proliferation of myosatellitocytes
E Hypertrophy of smooth muscle cells

Histological specimen of an ovary demonstrates a spherical structure composed of large


glandular cells containing lutein. What hormone is produced by the cells of this structure?
A *Progesterone
B Estrogens
C Testosterone
D Corticosterone
E Aldosterone

Atretic bodies and developed yellow body can be observed along with follicles of various orders
in an ovary specimen. What stage of ovarian and menstrual cycle is characterized by the
described ovary condition?
A *Premenstrual
B Menstrual
C Postmenstrual
D Regeneration
E Follicle growth

Histological specimen of the ovary shows large hollow structures. Primary radiating crown and
is situated in the cumulus oophorus, the wall is made of follicular cell layer and theca. What
ovarian structure can be characterized by these morphological features?
A *Mature (tertiary) follicle
B Primordial follicle
C Primary follicle
D Сorpus luteum
E Corpus atreticum

A microslide demonstrates an organ with its wall consisting of three membranes. The inner
membrane has tubular glands and undergoes cyclic changes. Name this organ:
A *Uterus
B Esophagus
C Vagina
D Ureter
E Urinary bladder

Which stage of ovarian follicle development is characterized by an initial period of follicular


fluid accumulation?
a. Graafian follicle
b. Mature follicle
c. Primordial follicle
d. Oocyte
e. *Secondary follicle

Which of the following is characteristic of granulosa lutein cells?


a. Are a minor cell type in the corpus luteum
b. Derive from the theca interna
c. Contain abundant rough endoplasmic reticulum
d. Are small and dark-staining
e. *Secrete progesterone

Which of the following hormones is primarily responsible for inducing ovulation?


a. Relaxin
b. *Luteinizing hormone
c. Progesterone
d. Follicle-stimulating hormone
e. Estrogen

Which feature is characteristic of the corpora albicans but not of atretic follicles?
a. May contain degenerating granulosa cells floating in remnants of follicular fluid
b. *Resemble large collagenous scars
c. Eventually removed by macrophages and replaced by stroma
d. Are remnants of follicles that degenerate before maturation
e. May contain degenerating oocytes

Endometrial glands are typically most fully developed and filled with product during which
day(s) or phase of a woman’s menstrual cycle?
a. Menstrual phase
b. Days 1-4
c. The day ovulation occurs
d. Proliferative phase
e. *Days 15-28
Which feature is characteristic of the endometrium’s basal layer but not of its functional layer?
a. Includes the uterine surface epithelium
b. Includes connective tissue
c. *Contains cells that replace the surface epithelium after menstruation
d. Relies solely on spiral arteries for its blood supply
e. Undergoes cyclic thickening and shedding

Most lipid in milk is released from cells by which mechanism?


a. *Apocrine secretion
b. Paracrine secretion
c. Holocrine secretion
d. Merocrine secretion
e. Autocrine secretion

A 42-year-old woman visits her physician complaining of recurrent vaginal yeast infections. The
doctor explains the likelihood that the woman’s vaginal lining is temporarily out of proper acid-
base balance, leading to the increased susceptibility to yeast infections. The normally low pH in
the vagina is maintained by which of the following?
a. A proton pump in the epithelial cells similar to that in osteoclasts and parietal cells
b. Secretions derived from intracellular carbonic acid
c. Secretion of lactic acid by cells of the stratified squamous epithelium
d. *Bacterial metabolism of glycogen to produce an organic acid
e. Synthesis and accumulation of acid hydrolases in the epithelium

You might also like